Download as pdf
Download as pdf
You are on page 1of 544
Preface This workbook accompanies the text Organic Synthesis: The Disconnection Approach. It contains further examples, problems, and solutions under the same chapter headings as those of the main text. I assume that you have read the corresponding chapter in the text before tackling the chapter in the work- book. Yon should then find examples and problems to help you understand each part of the chapter in the main text. Within each chapter, the problems and examples are arranged either to follow the same organisation as that of the main (ext OF ( present a series graded in difficulty. Particularly easy or difficult problems are so labelled that you can avoid them if you want to. My programmed text Designing Organic ‘Syntheses, Wiley, Chichester, 1978, provides another graded series of problems and solutions within a similar framework. My chief thanks go to Denis Marrian who gave me vigorous encouragement ‘as well as checking all the text, diagrams and references and providing index and formula index. His hard work and that of Marilyn Buck, who typed the printed words, enabled this workbook to be produced at the same time as the main text. Iam deeply grateful to them both. ‘Cambridge 1982 Stuart Warren Author’s Note Cross References: Cross references marked T, e.g. ‘chapter T 20 or ‘page T 145" refer to the main text, Other cross-references refer to this workbook. Index: The text and the workbook each have their own index, At the end of this workbook there is also a formula index of all target molecules in both the text and the workbook. Chapter Chapter Chapter Chapter Chapter Chapter Chapter Chapter Chapter Chapter 10 Chapter 11 Chapt 2 Chapter 13 Chapter 14 Chapter 15 Chapter 16 ‘Chapter 17 Chapter 18 Chapter 19 Chapter 20 (Chapter 21 Chapter 22 Chapter 23 Chapter 24 Chapter 25 Contents ‘The Disconnection Approach, Basic Principles: Synthesis of Aromatic Compounds ‘Strategy I: The Order of Events ‘One-Group C—X Disconneetions ‘Strategy Il: Chemoselectivity ‘Two-Group C—X Disconnections Strategy II; Reversal of Polarity, Cyelisation Reactions, Summary of Strategy A Strategy IV: Protecting Groups ‘One-Group C—C Disconnections I: Aleohols General Strategy A: Choosing a Disconnection Synthesis ‘Strategy V: Stercoselectivity ‘One-Group C—C Disconnections Il: Carbonyl Compounds . Strategy Vi: Regioselectivity Allkene Synthesis Strategy VII: Use of Acetylencs ‘Two-Group Disconnectionsd: Diels-Alder Reactions, Revision Examples and Problems ‘Two-Group Disconnections I: 1.3-Difunctionalised Compounds. and a,8-Unsaturated Carbonyl Compounds ‘Strategy IX: Control in Carbonyl Condensations ‘Two-Group Disconnections Tl: 1,5-Difuuctionalised ‘Compounds: Michael Addition and Robinson Annelation Strategy X: Use of Aliphatic Nitro Compounds in Synthesis ‘Two-Group Disconnections IV: 1,2-Difunctionalised ‘Compounds . . : : Strategy XI: Radical Reactions in Synthesis: FGA and its Reverse, Two-Group Disconnections V: 1,4-Difunctionalised ‘Compounds . a ” 0 9 96 107 23 134 14s 10 173 188 200 209 229 241 267 284 Chapter 26 Strategy XII: Reconnections: Synthesis of 1,2- and 14 Difunctionalised Compounds by C=C Cleavage Chapter27 Two-Group Disconnections VI: 1,6-Difunctionalised ‘Compounds . Chapter 28 General Strategy Bi Stesiegy of Carbonyl Disconnections Chapter 29 Strategy XII: Introduction to Ring Synthesis: Saturated Heterocycles ‘Chapter 30. Three-Membered Rings Chapter 31 Strategy XIV; Rearrangements in Synthesis Chapter 32. Four-Membered Rings: Photochemistry in Synthesis Chapter 33. Strategy XV: Use of Ketencs in Synthesis Chapter 34 Five-Membered Rings . Le (Chapter 35 in Synthesis: Chapter 36. Six-Membered Rings . Chapter37_ General Strategy C: Strategy of Ring Synthesis Chapter 38. Strategy XVI; Stereoselectvity B ‘Chapter 39 Aromatic Heterocycles Chapter 40 General Strategy D: Advanced Strategy References Formula Index Index 299 an 335 393 368 376 389 397 407 4a 29 492 470 491 si sor CHAPTER I The Disconnection Approach Two syntheses of ketone (1) are described in Chapter Tt. ‘The starting materials for the two syntheses are different: (2), (3), and (4) were used for the industrial synthesis, (5) and (6) for the laboratory synthesis, Analysie shows that the same bond (a) was nade in both syntheses. Industrial synthest. Laboratory Syntheste ‘ \ 9 ae CogEt ©) w) If you cam analyse published syntheses Tike this, you will increase your understanding of good places to make good disconnections Problon Two syntheses of multistriatin follow. For these and for the synthesis in the main text (pT 3) draw diagrams like those above to show which parts of multi- striatin came from which starting materials in each synthesis. Do not be too concerned about the details of each step in the syntheses. Synthesis! 2 acetal Me™" x formation x Ne ————> Ho. “Gponidation™ (ile gCub ) He ye wile 2 TSCL, pyr =n as H eH 2.Nal 0 4 mm x ol t ~*~ md 0 Bucy —_ (8) multistriatin Synthesis? 3 1.CH0, Me Ni HCL |) soma Quinoline, MeOH 290°C at (9) + 0)» — 15) : ‘never Tn each case, simply draw the structure (6) and trace which atoms came from which original starting materials. Syntheste + ABS wm Syntheata 2 @ Bond (a) is made in all three syntheses end bond (b) in two of them, We shall see later that the symmetry of ketone (7) is a reason for (a) and the branchpoints in the molecule a reason for (a) and (b). Syntheste 3 ~ N \ oO CHAPTER 2 les: Synthesis of Aromatic Compounds Disconnection and FGI zzanple : Steps im the synthesis of multistriatin given on page T 3 correspond to disconnections or FGis as follows:~ Se AG ‘This is a rearrangement and is best described as an FG1 since no new C-C bonds are formed and all carbon atoms retain the samo number of bonds to oxygen atoms. Could be deseribed as epoxide by oxidation) ots => : Definitely the major disconnection - a new -€ bond is formed. Could be described as a "> C-0 disconnection or as => an FG1-conversion of OH to OTs vy substitution => ©: FG1: COsH converted to cu,ou by reduction with LACH, It may surprise you that there is not always a cut and-dried anawer to whether cach step corrosponds to FGi or disconnection, but this is so, particularly when C-X (5 = 0, N, $ ote.) bonds are to be formed, Discussing the type of logic invelved in each step helps you to understand the logic of the synthesis : both aspects (FOL aud disconuvetiou) are helpful - choose ubichever seems the more helpful in the circumstances, Problem + Repeat this exercise for synthesis 2 on page W 2, Give each FG1 a full description (oxidation etc.) and label each disconnection C-O ete. according to the bond formed. Anewer : he before ~ a rearrange- —> ment best described as Me Fei. 8 ‘The major C-C dis~ = wt synthesis HO" H wt C-I disconnection or (vin OTS) yo FEL (alcohol to tos sylate to iodide by substitutions) x . Rearrangement : helpful on to call this FOL x + Me,cutiA C-€ disconnection ‘Two C-0 disconnections or FG1 (olefin to epoxide by oxidation). Ho" . Two C-0 disconnections * is the most berprur description. Avonatie Eleetrophilte subetituedon Feamplee : DDT (1) is a very useful insecticide but builds up in the environment with unacceptable effects on wild lite. A biodegradable version of DDT such as (2) might solve these problems.” OO “OO cen, cols 3 one m @ Disconnection of (2) at the bende joining the aromatic rings to the aliphatic part of the molecule will need a reagent for the doubly charged syathon (3). Anatyete ccly cul 4 Wiuadide Wild HUE Gu as Le wuuad ve unreactive towards substitution and we already have three halogen atoms in (3). The answer is to use the aldehyde (4) ~ chloral - in acid solution. One addition gives the alcohol (5) which dehydrates and reacts again via a carbonium ion. In practice (5) need not be isolated. Syntheste® Meo, Meo, > cu ond cuo, @ chy (8) Meo Meo, ou, > O., —> may éc1y ely As more and more waste products are re-cyciea, nethods of extracting contaminants from them are needed. Reagent (6) is used to remove boron compounds from alkaline brine wastes.* Analyste ou on oH cu (6) ro) Removal ot the one-carbon electrophilic formalde- hyde (ct pT 19) leaves a chlorophenol (7) made by direct chlorination of (8). The long allyl chain of (8) cannot be put in by Friedel-Crafts alkylation or re- arrangenent will vecur (p T 9) so acylation and reduction are preferred. Syntheote® on on — + och reduce acl, © 2 ono o—> a) ES HE) 90% HO’ H. C, Brown’ used ester (9) to test a new reducing agent which did indced form alcohol (10). How might he have made the ester (9)? copie Nabil, —s cr alcly cr (3) (10) 84% 0 dnever : Ester (9) can easily be made from acid (11). You might consider two approaches to this : a one-carbon electro~ phile addition via chloronethylation (Table T 2.2) and waidation or FG1 (Table 2.3) back to p-cblorotoiuene (12). The latter is easier on a large scale. The p- ehlorotoluene (12) can be made either by direct chlorination of toluene or by the diazotisation route (pT 12) again from toluene Analysts FOI oxidation Or 261 or ond i chiore- .) methylation o= FGI on oxidation © coh Ne ed => ( ort a) " diazottsat voll thes reduction Me™ FGL SI 1 on! Hy e Me Chlorination with a Lowie nei aatalyot? at Tow temperature avoids chlorination in the methyl group but the diazotisation route,* though longer, is perhaps easier.* Syntheago" wo lo 7 ay 101204 co,it => —=" . cr er separate from ortho compound ro, eon sys 28% 1.taNog C1 ,0°¢| 2.ca( Cr saa ™(9) Me e — 05% sn,ucr "aN separate from ortho compound Sinpie Probleme : For practice, especially for beginners. How would you mike TMs (13) = (15), phich were needed for the reasons given on the right? Pir rucaay geranium odour in soap? study of cyclisation reactions in acid solution?? * In fact, 211 these compounds, even TM (9), are available commercially. 13 exploration of NO, industrial uses of HF?! ome mcs) usione to Simple Problene 74 (13) : This symmetrical molecule offers only one dis- vonnection (a, below) corresponding to a Friedel-Crafts alkylation (pT 6 )« : sxalyede or0=-0"0-O aici, a 2s m3) Phu + Pacis TM (14) : This time the usual disconnection corresponds to a Friedel-Crafts acylation (p 77), =o . Any snalyede (14a) synunwote’* Pail + CY — ma) TH (15) ; Disconnect the nitro group and the alkyl group in either order. Anatyete Me b |lr-c c-clJarkyiacion ome Tu (15) has been made by nitration followed by Friedel~ crafts alkylation with IF as catalyst. No doubt the alternative route would be successful too. syntheste 1H tte te HNO, — SS assy Ny separate from para compound CHAPTER 3 Strategy I: The Order of Events General Strategy Examples (see table of guidelines) How that you appreciate the reasons why a particular order of events is followed, study the syntheses of benzocaine (p T 6) and piperonal (pT 9), The benzocaine synthesis involves a great deal of FGI : two of the four steps are Gls. This is because two FGs required for the synthesis (NO, and COjHf) are m directing and so cannot be used to cirect substitution where it is needed (p). (guideline 1 in Chapter T 3). Nitro is the group to put in last (guideline 2) and FGI solves the orientation problem (guideline 3). ‘The piperonal synthesis (pT 9) is an example where the two ortho oxygen substituents would be very difficult to set up. Me therefore (guideline 7) start with available catechol (o-dihydroxybenzene). Table 3.1 Guédetines from Chapter T 3 1, Examine the relationship between the groups, looking for groups which direct to the right position 15, If there is a cholee, disconnect féret (that is add last) the most electron-withdrawing substituent. If FG1 is needed curing the synthesis, it may well alter the directing effect of the group and other substituents may be added before or after the FGI Many groups can be added by nucteophétic substitution on a diazonium salt, made from an amine. Adding other groups at the amine stage may be advisable as the amino group is strongly 0,p- @irecting. As a last resort, there is a trick to solve some difficult problems, such aS adding two 9,p-directing groups, m is one another. A ‘dummy’ amino group is added, used to set up the required relationship, and then removed by diazotisation and reduction. Look for substituents which are difficult to add. It is often good strategy not to disconnect these at all, but to use a starting material containing the substituents. Look for a combination of substituents present in the TM and in a readily available starting material. Avoid sequences which may lead to unwanted reactions at other sites in the molecule. If o,p-substitution is involved, one strategy may avoid separation of isomers in that the other position becomes blocked. Guidelines may well contradict one another ~ use your judgement! fomanal. Stuategy Problema 1, Analyse the syntheses of (a) BIT (page TS) and (>) phenol (1) (pT 12) and see which guidelines * OH OE» Anower (a) 7 and 9 in particular (>) 3, 4, and 9 in particular, 2, Im the trifluralin synthesis (pT 13), what is wrong with this order events, suggested by guide- lines 2 and 9? aR, , RR, “, NOp CN — => ete nitration cr, 3 ie) Anover = nucleophilic substitution no longer possible = CF, m-directing, 80 difficult to make (2) (guideline 15 nitration might oxidise amine (guideline 8). Further Worked Beanpleo 1. In 1979, workers investigating!’ the dicnonc-phenol rearrangement treated (3) with acetic anhydride and got an unknown compound believed to be (4). Treat~ ment with strong acid gave (5), they supposed, but they wanted to synthesise an authentic sample of (5) to check their assignment. This 1s a tricky problen 18 as none of the substituents can be easily dice connected. ° ou > on , One 0 — —S 1,0 oF cO,8 pi Now " “3 ay (sy? We really would prefer to keep the two OH groups and start fron catechol (guideline 7). This forces us to disconnection (3a). We require a reagent for synthon (6) and the obvious choice is the diszonium salt (7). Analyste on 7 on x . i.) (1) + Phi 2 on on co on = " z ‘The OH groups must be protected against oxidation during the nitration step und the methyl cther (8) 4s convenient starting material. Treatment with HBr at the end of the synthesis, removes the methyl groups. frtonie’ . bated 2. Phi ‘The product of this synthesis proved to be identical with the unknown compound from the dienone- phenol sequence and the structures are confirmed. simple Probleme Suggest syntheses for Ts (9) und (10) needed as inter- mediates: TM (9) in the synthesis of brominated hydroxy- benzoic acids'* and TA (10) in the synthesis of model compounds for studying biological mechanisms of ester bydrolysis.7* ox 09H 7 O S 1HO (9) (20) oluttons to Simple Problems fu (9) With two electron-withdrawing groups, FG1 is cascntial to get some control over oricntation (guide Line 3), The easiest is COs to Cs. Then dis- connection of either Br or NO, is possible. Anaiysie £0 ear *s oxidation’ Ogi Br one a ® ap a de Hy Br a2) (as) ‘The nitration of (12) would probably give (11), but bromination at one of the two identical reactive positions of (13) is the logical route (guideline 9). Nitric acid oxidises the methyl group to COyl!. syntheste!® 37 (ity Cy Brg > qi) | T 100% «as) separate from ertho isomer HNO, (aa) ———> ™) He 1,0 76% An alternative synthesis of (11) relies on the di-nitro groups (p T 38) but the partial reduction of yield is poor. Syntheste 2°° L.NaNO, Me Sac © apr 0% =—> Pane?) HC 2.CUCD)BY” ago 2h NO, Onn i TW (10) We would prefer to keep the Ol group (guideline 6). Bither CHO (Reimer-Tienann, table T 2.1) or t-Bu (Friedel-Crafts) may then be disconnected as the orientation is correct for either (0, p- to the most electron-donating group -oH) (guideline 1), Guidelines 2 and 9 suggest disconnecting CHO first anatyede eimer— Friedel. Ht emana M Crates oy => ==> ‘CHO 2 (aoa) Syntheoto! S19 Ol ap HM cuciy ——=> —— > ma) NaOB 7 a? ose Mone Advanced Bwanpte 2, The tetracyclines are important antibiotics often effective against organisms showing penicillin resistance. In 1980, @ synthesis of some sulphur containing tetracyclines*® required thiol (14) as an ‘The SH group could be introduced by nucleophilic displacement of a diazonium salt (guideline 4) s0 amine (15) ie an eeeential inter- nediate, Analyate 1 ch a cr Ht Aly => Wet Med one as (as) We would rather leave the OMe group alone (guideline 6), the amino group will be added via nitration and reduction which gives us some flexibility of orientation (guideline 3), and the chlorine can be added by direct chlorination or by diazonium displacement. ‘The most obvious disconnection is to remove the chlorine, Un- fortunately chlorination of the very electron-rich amine (10) oxidises 1t to black tars : it would in any case give a mixture of isomers as all positions in the ring in (16) are activated. nalyata 2 cl 1 Nit, eblorination Ve Oxte (5a) os) One other possible short cut is to disconnect the amino group, hoping to add it by a benzyne mechanism from easily made (17). In fact, this reaction gives a mixture of three isomers of (15) in which (15) does not predominate.?* ome one ome one cn NaNHy Hy oe — + + NBg(1) BN HN a a 1 an sy We must therefore invoke FG1 and convert NB, into Noy. This is not immediately of any great help as neither of the obvious disconnections (18a and b) is any good. In both reactions, the strongly electron-donating Me group will dominate and reaction will occur ortho to it*'s?® (b) or give a mixture (a) Analyeie 3 (a) NO os FGI NOy os) ———> b Ole reduction cL . Tie By (1s) x ‘The chlorine can he disconnected with the alternative polarity via the diazonium salt to amine (19) (guideline 4). The amino group is more powerfully electron-donating than OMe so we can disconnect the NO» group. The amino group is itself derived from another nitro group. Analyeie & cl xi, N0y coon NO, diazoniui nitration salt OMe Me ue as) aa) (20) WO, FOL =—_ reduction nitration OMe e It will be wise to protect the amino group of (20) by acetylation to prevent oxidation during the nitration step (cf pT 12), Syntheete?? 2? Ho, Nil 0, Hy ,Pa,c 1.Aeg0 > — —_—_— 2.118 BNO, we Owe 0 (20) tine NOy 10% KON 1 HNO i, —?> (19) — 118) —> SHS) EtOW 2.cu(T)Cl a2 ote fore Advanced Problems 1. Polyurethanes are often polymerised onto rigid diamines. Compound (21) was needed for a new poly- urethane?" Suggest a synthesis for it. Me (au) 04% NO, ‘mower The nitro groups must be put in by nitration but the orientation is wrong. The methyl group cannot be disconnected as a Friedel-Crafts alkylation would never work on the available but extremely unreactive m-dinitre benzene, The solution (guideline 5) ss to introduce a dummy amino group ia such a position that 41 can activare the required positions. The best place is p to the methy1 group Analysis we We add on chapter o=> ===> ate NB, Og) NO, nitration 2 Oe 2 group Mu A protective acetyl group is again recommended during the nitration (cf p 25 and pT 12). Syntheste?* Me Me 90% HNO, chapter T2—> — oy NO hucowe Nac (2) Me 50% HyS0, KaNOy ty —> mar 0,6 “NO, Hy80, nee 0H 0% from (22) 2. payluectde ‘The dreadful tropical disease bilharzia is carried by water snails and one way to tackle the disease is to kill the snails with a moluscicide harmless to manmals and fish. Bayluecide (25) i made by Buoyer*® for this purpose. It is an amide, made by condensing acid (23) and amine (24). Can you suggest syntheses for these two compounds? Bayluscide : Syntheais Reagents: PC13,xylene,150°C on Op iH Xe, + => 1 gt By cr sf cL 0 a (23) (aay (25) ssid (23) ‘Tho combination of OK and COpHi next to each other suggests salicylic acid (26) as a starting material (guideline 7), Chlorination should occur 0, p- to the activating OH group and p- should predominate for steric reasons. This turns out to be correct inalyete on i => cv Og ‘Cosh (23) (26) ynthooiat® Oy (26) —4> m22) imine (24) Disconnection of chloro or nitro groups is satisfactory as the amino group will direct very strongly 2 gp. Outdoline 2 anggests route Ch) route (c). The published synthesis Analy. . guideline 9 uses route (b). © en 1 (a7) chloroaniline (27) is available nowadays but it can be made from the nitro compound by reduction.?* Protection during the nitration step was by a 'tosyl' (toluene-p-sulphonamide) derivative.?? Syntheote?? »?® Synthesta?? + steam distil from traces 0) 4 Te To WH, para compound cL 24m dilute acié coparate from para compound 9% 0, ss HN > 80% HORS —S> TH 4) ‘Tenet cL a1 1204 CHAPTER 4 One-Group C—X Disconnections se-noup OnE Bescomacstons Ho AeNaotl inpte Beanples -8cc: 2.€1,0.8C1 3 ni (2) Ho The garden fungicide ‘captan’ (2) 1s made trom (1) This is an imide - a double amide ~ and so both bonds between carbonyl and nitrogen can be disconnected dnalyote " a imide trcox (ay # (3) ‘The hest reagent for the double acyl derivative (2) is the anhydride (4) whose synthesis is discussed in Chapter T 17. Reaction of (4) with ammonia gives TM (1). Syntheete®?® (Ng) 5605, Ss may 30. In making esters, either the alcohol or acid may be in short supply. Precious acids are often treated with large excess of the alcohol (e.g. as solvent) with mineral acid catalysis. Meotl RCO,t| —————> RCO, Me # The potential analgesic (5) is an oster of the alcool (6), which has to be made (sce pT77), and acid EtCOgH, The chosen reagent was the anhydride which wastes half of the cheap acid but ensures efficient con- version of (6) into (8). Analyste Pa oe 8 ier Mey (5) Syntheate?#(4) pyr (6) + excess (BtC0),0———> TD) Simple Probleme 1, Mark all the C-X bonds you would wish to disconnect in these molecules and say why. H Intermediate in iso- Me Ph 2a ey quinoline synthesis Ph Meo" m aoe Benomyl?* fungicide for plants HCO Ke (8) 2 Pt 53-8, ~ y CX Anpicillin penicillin & ™ ¥ type of antibiotic (9) foun Choosing a11 the bonds joining a carbonyl group to a heteroatom (a) or joining a heteroatom (not Chapter T 8) to an alkyl group (b). Not all of these bonds would be disconnected in any one synthesis but it is important that you learn to see the possibilities quickly. 2, Choose starting materials for a one-step synthesis of ether (10) saying why you chose these particular (10) Anaver ‘An alkyl halide and an alcohol under basic conditions will give the ether Analyste MD aay (10a, b) Route (a) ie preferable since henzylic bromides are reactive towards substitution and cannot eliminate whereas the secondary halide (11) in route (b) is relatively unreactive towards 8,2 and can eliminate. Synthesis? So oO pase + mao) Howe advanced example Methyl damascanine (12) has a nutmeg-like smell and is used in perfumory. Each group on the aromatic ring has an alkyl group attached ~ other, amine, and ester are all present. CO,te NHMe aay we ‘The ester is the casiest to make so we ean éis- cet that first. The alkyl amine will be made by methylation of the aromatic amine (13) (see Chapter T 6) and we are left with 2 familiar problem of aromatic synthesis. nalysis 1 ayo Og ‘OH ste C-0 augue => => ester one He one (12a) a3) ‘The amino group mst come from a nitro group and Girect disconnection of (14) has the correct orientation though whother (15) will nitrate at eites (a) (b) or (c) is doubtful. Co, rot ie on ony ow os) => 0, —$—$—> reduction nitration % “OMe ‘onte aay «s) Experiments showed that while nitration of the available hydroxyacid (16) went in position (b) conditions could be found!* to nitrate (15) at position (a). Disconnection of the methyl group from the ether must therefore be the last step in the analysis. Analysis 3 0,81 0,8 Bs Me ether ow une 0, 9804 on Ko, 2 (18a) as) Dimethyl sulphate (p T a1) is used for the ether synthesise, and oster}fication was with oxcess methanol (of page 30). Synthas a COgit (Meo) 980, «Ko, —> > NaOH 180, ‘on le a (15) 09% ¢0,) 10g =—_. ay Me ‘Ove 2 HM yen —_— maz) HCL ‘ome Problem Suggest a synthesis of the local anaesthetic ambucaine (17). This time carry the analysis of the aromatic starting material further, for revision.* Alcohol (18) is available (and is discussed in Chapter T 6). HO, ™ate, cas) MH, 2 a7 Disconnection of the ester removes alcohol (18) and leaves (19). ‘The amipo group will have to be put in by nitration and we already know (p 34) that we need the free acid (16) for this. The ether must therefore be disconnected first. + oxidation of 0, m, or p-hydroxy toluenes ie not a way to make the corresponding benzoic acids as other reactions occur: Analyte 1 GOH co ap —> ust as) ester hu ‘Opt c-0 — eS cays tune reduction SN etter Xo, 2 ‘Oght GOK co => nitration on 16) (20) “Nog as) The CO,H group is correctly meta-directing in (16) 50 the best route will be via a diazonium salt back to benzoic acid, Opi Analyata 2 co For oe) diazonium reduction Wi co,i zs 09H cn —=> Oy Experinents*® showed that nitration of benzoate esters (e.g. 21) is cleaner and the diazotisation’* is straightforward. as an este The acid group in (20) was protected v during ether formation and the acid chloride method was used for the final esterification. 1.0, Gogtic BHO. cova Gogu 2 11,804 2 Hy ,Pd ® = Nao > 3.HCL °. , (a1) 83% 2 2 0,1 co,u 1.1lN0, ay Ei 2 epatwt O,heat “ar? (16) Co,Et a) 0g 1. Kol 1. xou ———_— i ene ‘oBy 2-SOC1, No, ho, % ain 2 60% coca 1.18) 17) ‘oBu 2H NL 63% Xo, 2 95%, CHAPTER S$ Strategy I: Chemoselectivity Bramp! Ketoacid (1) is an isomer of a compound made in the book (p T 39) and both were needed for the synthesis of potential fungicide: 1 Direct Friedel-Crafts disconnection is na good Preliminary (p T 39) as it led to the other isomer. €-Cl disconnection is the answer, with anhydride (2) providing the chemoselectivity. Analyste cn c-cr === chlorination cog SMOFARAE CoH ay ° Friedel- Ht =—=—- ‘0 Crafts @ 38 It turned out that the Friedel-Crafts reaction and the chlorination can be done in the same pot. The vhlorination needs to be chemoselective as reaction on the methy! group oF next to the carbony? group could secur. Lewis acid catalysis is the answer, aoate?" 1,(2), ACLs 10°C Sa EEESUEENeT EC auGe) 2.C1y,ALCLs 30°C 16.36 9.26 & Problem Shy are the chemoselective reactions (a) and (b) successful but (c), used on page 21, unsuccessful, ca) LaDy 00a 0,82 ——> siela H oF u The product was used in a synthesis of some specific H inhibitors. ?* 2 ° 09) good yield Ke “ about SOL a * «© —a > ogi Oy ony" My Anawers (The guidelines appear in Chapter T 5) (a) Esters are more reactive than amides so the right reagent, or in this case LiBHy, does the job. (>) Amines attack acid chlorides easily as the amide formed is conjugated, The alkyl halide is notably less reactive. ‘These both fit guideline (1). (c) The nitro groups are too nearly equal in reactivity for a distinction to be made, Had they been identical, all would have been well : the problem is that they are nearly but not quite identical (guideline 7), Move Advanced Esanples ‘The monoketal of some diketones is formed in reasonable yield by the statistical method. Thus (3) gives a reasonable yield of (4) : after separation from (8) and the bis-ketal, 64% of (4) can be isolated.** wd uw oreo om > 5 q x 3 two-phase = © system “ However, it does not follow that in any individual case, the same applies. The monoketal*? (8) of the cyclic diketone (5) is much in demand as a synthetic intermediate and in conformational studies, Mono ketalisation is unsatisfactory here - very little of (6) can be isolated. This may be simply because the right conditions have not yet been found. Various solutions to this problem have been published*?: I shall describe two, both based on the statistical method. § ry Oxidation* of the diol (7) with one equivalent of chromic acid in acetone gives an excellent yield of (8). ‘The rest of the synthesis is trivial, as the two functional groups have been differentiated, ntheete 1 ou rm e xf \n cro, > > errr @ y (7) de cs) OF oH ‘The other approach’? uses the monoalkylation of guinol (9) as the chemoselective reaction. The rest of the seyuenceis more advanced but the principle should be os ote Ole eS SO-3 wet Ae => & 3 (20) An advantage of this metuod is that separating acidic (10) from the neutral dimethylated by roduet is very easy. Probleme 1, How would you make the half protected aldehyde (12 given that (11) is avaflable from the reaction shown XK, (cOMe a) et CHO Olle qa) Anover : The two Tunctioual groups are wlready differentiated, it is merely a matter of FGI. One solution is: Syntheate*? MeOH Me ‘ole LAAIM, oy— a“ 4 Me es Pee SYN Ss m2) ute ‘The acetal is used as a protecting group during the reduction of the less reactive ester (guideline 3). The product (12) was used"* in a synthesis of gascardic acid. 2, Suggest conditions for this synthesis of the analgesic alclofenac, aay aay sewer : We need to remove both acidic protums frum CO,H (pKa ~ 5) and OH (pKa ~ 10) then ArO™ will be the stronger nucleophile (ef p T 34 ) Suntheete"® K,C0,, acetone (13) Ss tury aa 3. Suggest synthesis for (15), needed for studies on oxidative decarboxylation. *€ ° 1OgBt as) “ewer : Friedel-Crafts disconnection requires the halt ester, balf acid-chloride (16) which we could make from maleic anhydride (17) as on p 739. An easier solution 48 to use the anhyuride in the Friedel-Crafts reaction The two methyl groups cooperate to activate the right Position and the yield is excellent (OgEe a ic : — J? rrivuer Ko >_> ae) «sn 4 ! 44 oyntheste®? ° aici oH Exon, ut 3 = —> mas) ary 915 CHAPTERO Two-Group C—X Disconnections 8) 1, 2-Difunetionalieed Compounde ‘cognising an acetal is a problem for many chemists in training. Molecule (1) is obviously an acetal, and the first disconnection is easy to write:- ssn, CO. oe Both starting materials are available, the glycol 2) being a cheap and efficient acetal forming reagent. CL oN —= TM(1) # Recognising the acetal in frontalin (2), a pheromone of the western pine beetle,** Nevertheless, it is important to look for the two oxygen atoms Joined to the same carbon atom (@ in 9a) and dis- connect the acetal before considering any other steps. Jentheata is not so easy. 46 fo Becounisg = «3 OH Intramolecular acetal formation is so favourable that any attempt to make (4) leads at once to frontalin, Recognise the acetal in the following Tus and carry out the first disconnection. Pe Jasmine odour compound’? DB 3) oynthetic intermediate’® anever : The acetal carbon atom is marked @ Ph Ph yw a teaix, \ ee? Smt ud ba + c,0 Ga) out 1 teaix k HH => peti « acetal es a =glycerol 6a) ° ot 6 Problem : Suggest a synthesis for the perfume constituent (8). «sy ‘mover ; There are two acetals here! Tither could be disconnected first, though not both, since problems of chemoselectivity would arise. Since we have already made piperonal (9) (page 7 9) we shall use that as an intermediate. fnalyste Et0y, Ot cHo recognise 2 Leaix (8) acetals acetal oy cumpter erred 2 acetal 12 ot og oH available oo catechol Acetal (10) is available, though expensive, and is made from catechol and CiigX, (X= Cl, Br, I) in base!? instead of the usual method. The final acetal is made by the usual acid- catalysed method, using the ortho-formute HC(OEt), a8 a debydrat ing agent .** SyntheateS? $3 ‘sed Compounds (b) 1, 8-Défunceton Ssaupte : Many antiseptics are quaternary ammonium salts with detergent-like action and Benzethonium chloride (11) is particularly efficient, The benzyl group can be added by alkylation of (12) (Chapter T 8) which contains two sete of 1,8-diX relationshipo. snalyeis 1 ce py : Mey Oy NANA (1) > "ey! (2) Disconnection at any of these positions is possible (12a), We might preter a central disconnection (b or c) first (see Chapter T 11) but we shall end up with Me,Nn, two molecules of ethylene oxide, and the phenol (13) as starting materials. Analyte 2 enn + 2 Hep + PY A (chapter 76) en (12a) Q a. Ho" «sy ‘The phenol is clearly a Friedel-Crafts product and the required cation (14) ig most conveniently obtained from 'di-isobutylene' - a mixture of (15) and (16) available industrially from the dinerisation of Meg C = CH. Analysis 3 » Ske ot = Sor . ‘HO’ HO a4) (asa) ya Ak Lk (1s) 6) ‘The patented synthesis'* combines the reagents in a eifferent order and relies on statistical control of chenoselectivity in the reaction of (37) with (19). The advantage is that syimetrical (17) nay be made trom ethylene chlorohydrin. cynthestes* "55 pnow ——25)* 46) 5 (45) aici, low temp. HyS0, az) Me NH Paci —— =a) ———> may vobleme 2 ‘Two problems in this section use sulphur nucleo- hiles as they are not so well represented in the main ook. Alcohol (18) is used in the manufacture of demeton, a family of systemic insecticides,** How can it be nade? Analyte 1, 2-uio, R Ets OAs Est (asay Ethylene chlorohydrin in base can be used instead of ethylene oxide Synthesis > mus) BtsH— Fe ‘The related cockroach repellent (184) is made from a thiol with ethylene oxiae.** aoctst + ZY —> joer" (aa) 2, Intermediate (19) was needed for a synthesis of the thianaphthalene’? (20). How would you make (19)? oo oO «a9) (20) Anaver : Friedel-Crafts disconnection reveals 1,2-dix compound (21) easily made from thiol (22) and chloro- acetic acid, ‘Thiol (22) can be made by the thiourea route (p T 37). Feiedeloy, Gate cr (22) 1. thiourea, cr c0 Ht 1 EES (22) => 2.110" .H,0 5 ?,0, (2) <> 1018) 100% benzene 60% 3. Suggest a eynthesic fpr the anti-Parkinson drug orphenadrine (23). NMey Pa (23) Anauer : Disconnection of the ether reveals a 1,2-dix compound (24) and an aleohol (25) eaeily made by reduction from a Friedel-Crafts adduct (26). Analyote To. wes ct Kio, PO * Hey ey => = ao Nate, rer 2 reduction Pn (28) Pn (as) Either Friedel-Crafts disconnection will do, but o-toluie acid (27) is available’? from the oxidation of o-xylene so that route is preferred as no separation of 2, Brisomers is involved Syntheote®?)*® oOo, — . OAc (21) 77% ‘AlCl; Phit s0cl, — (28) > (es) —> ‘coca The reactive benzylic chloride (28) is used for the final other oynthesis, ©) 1,8-défunetionatiaed Compounds sanple Compound (29) was needed‘! tor photochemical syelisation to (30), The obvious ether disconnection tooks promising but experiments had shown’? that it was alfficult to alkylate (31) without polymerisation (29) (20) 94% alyeta 1 AA aN AX Vy ner? Br + 10" (208) cay The alternative 1,9 4iX disconnection (29) requires the addition of allyl alcohol to enone (32) and this vorked well with BF, catalysis. Analysis 2 (29d) or Synthests®! (32) Da tem Probien A. Find tne 1,3-@1X disconnection in Tus (3% - 35) (33) 6 (34) “25 Lm jane, : c q (34a) The last vue requires u 1,1-d2a disconnection of the acetal before the 1,3-diX disconnection can be nade. te + MeOH (35a) What compound is needed to form cyclomethyeaine (37) from the intermediate (36) we synthesised on page T 35 and how would you make it? C0,H NNR oO} Alcohol (38) is needed, and it can be made by the logic of 1,3-4iX after FGI. (36) (37) snalyeds For AR, HNN reduction 210,07 2 38: (38) 1,3-dix SF 10,07°S vr, Lia, xR, Bloc“ + Ryu —> nlojo M2 ts mca) This is not the only routet* to compounds such as (38). The Mannich reaction (p T 158) provides alde- hydes and ketones (39) which can be reduced to (38) and its analogues or RjNH can be alkylated by the chloro- alcohol (41), available by the statistical method’* from eynmetrical diol (40). synvhes 8,0 NR, 0. mR, 2 2 2 ww a ~~ ~ HWR, Nautly 1 (39) (RtsH ar, ani) TN, HL CNA, (b) ko’ ‘on ——> cr ‘oH ———> (38) «409 any (8) Generat Problene 1, Amino diester (42) was needed** for cyclisation and decarboxylation to (43), (method of Chapter 19) a useful synthetic intermediate, How would you synthesise (42)? 29 ° n-BuyN 1.Eto7 co,ee dscarboxylate (43) 79% (42) snover : There are 1,2-dix and 1,3-diX disconnections to pe ade. The order is not very important as the starting materials are the same and the order in the synthesis will be determined by experiment, sealyate 13 2 neBuN7 NF 1, gaa 3 SR tontt 2 n-BuNit te 7 = > + ‘Oyke 2) coo, (42a) 1, 3-dix n-BUNK, + Qi -CO,Et ‘This was the order followed for TM (42), but in other cases, the Michael reaction came last syntheote®® Akt 88% 1 BUNH n-Buni CICH{CO,Bt ne uy 40H > maz) C05 90% 2, Propose a synthesis for the naturally-occurring amino acid methionine (44) using the logic of this chapter. ue ~Nco H 44) Ansver : The Strecker amino acid oynthoeis (p T 43) requires aldehyde (45) for which a 1,3-diX disconnection As appropriate. Analyste Bo Strecker 39 ho — vee Pog TE yuh (44a) (4s) 1,3-48x Sess o5 This synthesis is very successful and has been used as the basis for commercial production. wesit (MH, 9605 Aci). ——> (8) — > ios eX not —> maa) My 0 cas * (46) was actually formed a5 yg ny 1 CHAPTER 7 Strategy III: Reversal of Polarity, Cyclisation Reactions, Summary of Strategy simple Probleme 1. ‘The last stage in most syntheses of the juvenile hormone (2) of aeeropta is the epoxidation of (1). For example, NCPBA (p T 52) gives’? 40% of (2) together with 10% epoxide at double bond (b) and 10% dicpoxide (a +b). No epoxidation oceurs at (c). Epoxidation via the bromohydrin®? is even more chemoselective, giving 52% of (2) Suggest some reasons for the chemoselectivity. 1.RORY #0 @ 2 2.1-Pr0 MCPBA : Bt,0 ae @) Anower ; Both epoxidations involve electrophitie attack on the double bond so attack at the electron-deficient bond (c) is unlikely. Preferential attack at (a) can only be steric as (a) and (b) are identical electron- jeally, In polar solvents, (1) probably coils up so 62 thar hand (h) 4e amhodded in the hydrocarbon chain and hot accessible to attack.7* 2. Oxidation of (9) with PD (ORC), was thought”* to give (4), Suggest a synthesis of this compound so that 4) may be compared with an authentic sample. Oo = 3) (a) 749 S Ensver : Various sequences of C-8 disconnections would Ro doubt all lead to good syntheses. Disconnection (4a) has the advantage of giving a synmetrical dithiol (5) and available (pT 53) ehloroacetylchloride. CPCS ‘s ‘SH er Syntheato”® Js . (3)-——> _ may ynis was indeed now (4) wus muge Lou Ho experimental details are given and the reaction went in ‘very low yield” Nowe Advanced Problems 1. Compounds such as (6) cannot be made by the opening of epoxides. Why not? Suggest a way round this difficulty Analysts (no good) 2 o® ow Nu, c-0 1,2-aix oo sw: R R (8) m Nu” = nucleophile, such as RO™, RS Ri, Ananar : Attack* would occur at the wrong atom of the epoxide, Nu NN Possible solutions include changing the oxidation level by FGI to (8) or reversing the order of operations 50 that R'O™ acts as the nucleophile. * considered more fully in Chapters T 6, 10, 12, and 23. Avatyete 1 on For nu, > ata = oe reduetion A R (7a) 6 ynthests 1 Br core Nun LAALH, — «&) —> R dnaiyete 2 Br, o=> R @ on? mo —> mo) «) * ao) Note that it ie necessary to open the epoxide (9) and substitute to get (10) - direct attack on a double bond would give’? the wrong isomer (11). Inversion of Bry on es ay alow R polarity (p 782) fe vital 2. Considering these strategies, how might you synthesise the compounds (12) which are used to inhibit the germination of weeds.?! Me ar ‘1 aay Answer + The chloroacetyl group can be disconnected first to leave (18) - just the kind of compound we have been discussing. The strateyy used in the published synthesis’* was the second of the two we considered. Anatyete OMe attack ccoura cleanly With Moo” ae nueleophir at the right atom in the epoxide,”® the bromide is made”* ‘ith PBrg and the synthesis finished according to plan. enthoste?*=7# ole oy one Meo- "5 ye es MeOH” uc be 63% 58% 013) ———— 1 1212) “yelteavion ple On page T54 we use disconnection (a) to devise a Since cyclisation is synthesis of morpholines (e.g. 14). 50 easy, we might consider the alternative double C-! disconnection (b) and use the bis-chloro ether (15), nade on page snalyois a i on 1,2-dix, ,_ => S SS om, A 0 QF o Nb mL aay as) ‘The cyclisation does indeed turn out to be easy : no intermediates can be isolated. syntheste** PANE (53) —$=—> mrs) Probien Carry out similar disconnections on TH (16) and suggest how the required reagent mignt be made. Ph Ph bn (16) Angver : There are 1,2-41X and 1,3-a1X disconnect ions to be made, giving (17) as the starting material which could be made by the bromination of enone (18). Anatyets ” . p . ast, 2-diX A ” —_—— Ph Dy a bri, 3-a2x ep ba Br an (162,0) ‘The bromination was carried out with N-bromosucein1— side (NBS, 19), a source of molecular bromine, and the -yelisation proved as easy as expected. unthesie?? PHRHy cas) + “Br ——> (17) ——4 > m18) 100%, «sy CHAPTER 8 Amine Synthesis Simple Example Amine (1) was needed to study the stereochemistry of alkylation reactions. The primary alkyl group had best come from an amide or an imine while the secondary alkyl group must come from an imine. The disconnections may be carried out in any order. Anat ° Pncocd A x o A ff reduction a (a) @ A + one synthesis’® uses the imine method for the second stage 20 Ms, A PHO > cK oy Te Nn, TsOH (2) 52%, Lialily vn on | ———> may 74% Simple Probleme 1. Suggest a synthesis for butam (3) a pre-energent herbicide Pay" «3) smover : Disconnection of the amide reveals secondary amine (4). ‘This can be distonnected in various ways Using reduction of amides (benzyl side only) or imides (either cide), Again the order docan't matter. . 9 A ak amide’ «aay «) F61|| reduction wy oN NN SK a Pncio + Pneoc + erm, One published synthesis" uses the first route with catalytic reduction of the imine, synthoade®® mem 1X 3 [eh JEM 5 co 73% oc 2) — eg 2, Suggest a synthesis for amine (5), needed to make ‘the potent neuronal excitant a-kainic acid,** 2 If you cannot see how to do this, look at Chapter TI and page T 65. er = Reduction routes are risky e.g. from (6) or 7), as we might reduce the double bond too. chem, ko (6) o The best bet is to use a reagent for the syathon “Hig” and the available chloride (8) (p T1). slyets ‘The published synthesis uses the phthalimide cethod on the corresponding bromide. nthoatot® 0 ’ oa a rY a 1.0, x01 7uC5) The next stage in the route to a-kainic acid was the alkylation of (9) with allylic halide (9). Even in ‘this favourable case (the halide is allylic) only 36% of (10) was isolated and "the low yield was caused by uncontrollable production of large amounts of dialkyla- ‘tion material", Well, you were warned! K,00, H,0 AX + oF cost Fy a o AAnrAon (oy Frampte Phentamine (11),used in the treatment of obesity, is a t-alkyl amine, The Ritter reaction, using HCN as the nitrile (p 763), is ideal for this. Analysts Niky CoN Ph’ —> wO>*K - MH, an oe synthesie'* i icy HCHO HOT a? —> ww Ss cay 750, Bp tote Len Phentamine analogues, @.g. (12), are also useful drugs. Suggest a synthesis. H BAN, Met (12) anauer : ‘The n-propyl eroup had best come from an amide (13) which could be the Ritter product if BtCN is used Making (13) by more conventional methods is fine, too. i FGI i MN — : rts (aa) 3) =. , Diborane was uscd in thie reduction, but LiAlly should do just as well entheate" Eten HOKE 1,80, x a Pally TF? C2) eo" 80% (13) 42% 76 Mowe Adnannad Renmpla Diamines find considerable use as pharmaceuticals here is one example followed by two diamine problems. Histapyrrodine (14) is used as an antihistamine. ab Phy yp, © aay Disconnecting one of the C-N bonds around the tertiary amine will help : (a) is aot possible, (b) would be possible after FG1 to the amide but (c) separates the two nitrogens and requires the easily synthesised (16) as intermediate. We shall follow (c). Analyste Ph, Nie Ph ¥ cas) (4c) = a Ho Q S25 457 oOo 9a Some reductive route (via amide or imine) will give (15) and the rest of the synthesis is simple. The final Wylation ie good ae (11) to crowded and unlikely rt a quaternary salt. Phill + PoHO —> paN-ciPn ————> (15) as) ¢ % v4, (6) ———> mas) 2.5001, roblene 1, Diamines (17) are used in the manufacture of armaccuticals and agrochemicals, ow might they be rade? antl, az) Snover ‘The primary amine might come from an amide or a cyanide, e.g. (18) : a 1,3-dix disconnection (reverse Michael) is then excellent. sealyete . FGI cu Sty reduction cas) SEES a + cas) Amines** add very cleanly to acrylonitrile (19) and catalytic reduction of (18) gives TH (17). 78 Suntneste 4, a9) 2 cat x > 6) ———— >a or Na/EtoH 2, Gabamide (20) 1s a psychotropic ugent. Suggest # synthesis? ; eNO (20) Answer : The primary amino group must come from a nitrile (21) and this time the Michael reaction is reversed : ‘1 Fol tg Ee Ne Ny reduction (20) (ay 1saix SS w+ Aron, Catalytic reduction is best for (21) > (20). Chenoselectivity in catalytic reductions depends solely on thernodynamie stability and the amide is much more stable (conjugation) than the aitrile hence the Neellent chemoselectivity. The acid is to ensure the free amine (20) is not formed or it might poison the catalyst cptheate®® Hy, Pt, KON DUE Arco, Ss on) 2 2 ‘TM(20) THN ae Bom a 100°C HCL ne CHAPTER § Strategy IV: Protecting Groups Now Protecting Groups Now protecting groups are always being invented : the improvements are usually ease of addition, selective stability, and most of all, ease and selectivity of removal. In 1969 a new carboxylic acid protecting group, the ester of alcohol (1), was introduced. cor oN 2 a@ The reagent (1) is easily made from thiol (3) and ethylene oxide or 2-chloroethanol (2). The acid RCO,# is then protected by acid-catalysed ester (4) formation. An om ~ RCO,H Jw, a) —>> Naor Soe oom “) 80 The reaction for which protection was needed, say +R, is now carried out and now the protecting group must be removed. This may be done by an elimination reaction (6) on the oxidised compound (5) under mich milder conditions than normal ester hydrolysis, as the p-nitrophenyl sulphony1 group is strongly anion- stabilising. 0.1m KOH 9 tok 4, 5. ss ()aetone? Oe Wt CGueois acotone Os co) Sadho he ahs As w i d Problem Amines might be protected as the amides (7). What reagent would you need to,put in thie protecting group, and how would you make it? The product amine (8) might be released by treatment with zinc. Suggest a mechanism for this reaction. ° RN, ——> ani —— @ 2 zn 1 Syn, + COp + AW SD im, + C0, + chy a) Anower : The chloroformate (9) would be needed. Ester Uiseounection reveals available 2-Jodoethanel and COC1y: snaiyets ¢ how atts hk (7a) ©) c-0. 1 = AL um ester Chemoselectivity is all right in the synthesis of (9) (p T 36), and the protected amines can be made from (9) with base eatalysig. This reaction is also chamo- selective as amines prefer to attack carbonyl groups to make stable, conjugated amides rather than attach Sp® carbon atons. Synthesio?® 1 01, rly ww Ss KrwXr bw (9) Naoit y i se of Protecting Groups imple Probleme 1, What unwanted reactions would happen during attempted oxidation of (10) to (11) and reduction of (12) to (13). How might they be prevented? on (a0) on ay aibydroxyeitronciial used in perfumes Meo, Cone faarn, MeO tt ») —_ Meo © Meo 9 (43) used in a tetracycline a2 antibiotic synthesis ver : (a) The aldehyde in (10) would be oxidised to COnH. Protection by acetal (14) formation is the ome Kiln, Mook 0) —_ HCL aay ome ¥ Me > aD HK #0 A 2 (b) The ketone will be reduced : again acetal protection is the answer.°? Note the use of the cyclic ketal (15) and the dehydrating agent HC(ORt),, ethyl orthoformate, both giving higher yields. Meo, ‘OMe Bs ad Yon Luan, (12) ——> — 115) HCCOEE) 9 ¢ 2.4 TH,0 mee LY as) 2. How would you carry out the conversion (16) + (17) the first steps of Marshall's bulnesol?? synthesis? as a as) an suer : Ether disconnection (17a) mist be on the side ssay from the aromatic ring. Alcohol (18) is made by reduction of the ester group in (16) and this will squire protection of the ketone. rtyste do = 4-4 cava) as) ‘The cyclic ketal (19) was again used as protection and the methane suipnony1 (MeS0, or Ms, mesylate) group was used as the leaving group X (p T28) (20), The protecting group was left in until the end to avoid any side reactions. ‘untheate?® wn, B (16) 24 COgEL am ‘Oar The p-chloropheny1 group is in fact itself a Protecting group which was not removed until a late stage in the synthesis. Mowe Adnannad Reampla In the synthesis’* of the A-chain of insulin, 21 amino acids mist be combined in a known order. The first steps use a protected dipeptide (22) and protected cysteine (21) ‘CHyPh k i 20, a Aone py He Hoo KO, (22) jy ‘The only nucleophile is the free amine in (22) and the most reactive electrophile is the p-nitro-phenyl ester in (21). These combine to give the tripeptide (23). caps e x i (21) + (22) ——> Pw if "cote (23) ‘The next peptide bond must be made to the ringed nitrogen atom, so treatment with HBr frees this amine without affecting the other protecting groups (table T 9.1). CHP Br xO (22) ——> N, WOKe” =H coe (24) Reaction with another suitably protected p-nitro- phenyl ester now adds the next amino acid (another cysteine). Another part of the same insulin synthesis involves this sequence. vice OE 25) Gly-He smigtfsoye owe i Pw (a) Explain the chemoselectivity of (25) + (27)? b) Why was catalytic hydrogenation used for (26) > (28) while HBr was used for the same reaction on (23)? (e) How would you free the terminal NH, group of (29) to react with another activated ester (e.g. 20), or activate the COpHi end to react with another free amine (e.g. 24)? (a) Hydrazine, SHjNHy, is a good basic nucleophile and will attack the most electrophilic carbonyl group available, In (25) the methyl ester is more electro- philic than either of the amides. Diazotisation then gives the aside : a, Ss thas ‘NH-My (en () The t-butyl ester protecting the side-chain of Glu fn (26) would be hydrolysed by acid (p T 71). In (23) ‘the S-benzyl group on the cysteine side chain might be cleaved by hydrogenolysis. (©) Use Hy-Pa again for the N-terminus as the t-butyl ester is still present, and hydrazine and diazotisation again for the carboxyl end. Selective protection and activation of amino acids and peptides has now reached a highly sophisticated level and requires specialised knowledge for the most efficient use . CHAPTER 10 One-Group C—C Disconnection—I: Alcohols Few carbanions are stable enough to be formed in solution a5 genuine intermediates by removal of a C-ll proton by base : “CX, (X= Cl, Br, T) are examples. The ester (1) io used as a rose perfume.** Disconnection of the ester reveals alcohol (2) which can be disconnected to “CCl. and benzaldehyde. 2, cl, “els Aa Sg ae Ph’ ester Phi "OH PhcHO a " The carbanion is mide from CHC, with KOH in the Presence of benzaldehyde. ** cHeL cHcocl PhcHo —— > (2) ———> ma) KOH Ro. >. Mone tynieal examples are the amino ethers (3) used as antihistamine or anti-Parkinson drugs according to the substituents.?7 These are obviously derived from the alcohols (4) which are made from an aryl Grignard veagent and a benzaldehyde. Either starting material may bear the substituont X : the choice can be made according to availability and so that side reactions are avoided, Analyeds z x ‘The ether bond is best made from alcohol (5) and the reactive benzylichalide (6). synthes? . 1 1.Me SOC Ly artsr, —————> (4) ——> 2. Ar*CHO 1 6) ° x a (6) MoyX¥———> (5) E> TMS) Probleme 1, The alcohol (7) was used on page T 30 in a Friedel- Crafts sequence. How would you make it? on an on m anaver ‘Two of the substituents are the same (n-Pr) so they can be added as n-PrigBr in one step Anatyete t (vay Syntheete?® Mg a-PrBr > (7) 2.Meco,Et 2. Aldehyde (8) was needed for a butenvlide synthesis. How would (8) be made? cuo oO” > Return to the alcohol (9) and disconnect to the Simple cyelopenty1 compound (10) and ethylene oxide snntynte 9 vat x A (ye => 7 onidation Taee oe a) PCC (table T10,2) 1s the preferred oxidant here. Syntheste?? ie poo an «) STH) oa 72% from (10) foide Problene 1, Acids (11) and (12) were both made by Grignard addition to CO, rather than by cyanide displacement (pT 60), Why? ay az) Ansver Cyanide displacement is impossible on halide (14) and gives much elimination on secondary halide (13) ‘The Grignard addition avoids these problems and gives high yleqas.'** 1g, Bt 0 pes man) 2.00, 86% as) Lalg,£t,0 Me0- sr > mz) 200, ozs 4) =. Suggest a synthesis of acid (15) used in an vestigation of oxidation reactions. TT (as) ‘The cyanide route via bromide (16) back to bromide 17) would be one obvious method, but the alternative sisconnection of two carbon atoms from (18) is shorter. ‘NS FSD DH | as) ret nied] ae 1,2 c-¢ “Y ee cas) az 94 The shorter route gives good yields : AgO was used as the oxidising agent.'*? syntheatet?* 18% 1 50 a TP 18) SD mas) o 70% 100% a 2, Suggest a synthesis of acid (19) used in an investigation of steric hindrance.?** oH as) Anever Grignard addition to CO, mist be the answer with (uis crowded acid and this leads us back to alcohol (20) Disconnection of any group now gives a simple ketone and an available alkyl halide, As we shall see in Chapter 11, it 4s best to divide the molecule into two nearly equal parts, so we shall use disconnection (a). (19a) on ats 2 Nie => ~,. ~~ (20) This route has been followed with reasonable success, though the yield in the carboxylation step was poor. Synthesis! 0,108 Le cL NO 020) —> aErcoe” “p 64% a teat AR SES 20) 2.00, 28% ey CHAPTER 11 General Strategy A: Choosing a Disconnection ‘The guidelines for good disconnections suggested in Chanter TL are: 1. Greatest simplification (in middle of molecule, at branchpoint, rings from chains Symmetry. High-yielding steps. Recognisable starting mterials Eeample : The synthesis of the natural product citronellol (1) (used in perfumery) shows guidelines 1 and 4 in action. Disconnection at the branchpoint is possible (1a) and the required alcohol (2) comes from available ketone (3) (pT1) by reduction. anatyote AnrK An aK, J&A on @ © 96 97 NaBH, PBrs «@) —> 1 @) — hat AwAL omy Br 8 a ‘voblem : Does the synthesis of miltistriatin on page 3 follow these guidelines? insver ; The main disconnection (corresponding to step 10 + 12) is chosen because it is at a branchpoint and because it uses the symmetry of Et,C0, In addition all steps were high yielding and compound (11) could easily be made, but you could only guese at thie from the chart Froblem : From the syntheses in Chapter 10 pick examples following each guideline er : Every synthesis uses guideline 1, We have branchpoint disconnections (all TMs), ring-from-chain diseonnections, (TMs 8, 11 : 12), disconnections towards the middle of the molecule (TH 19). Guideline 2 is naturally less general but Ts (7), (8), (1), and to some extent (3) all use it. Every synthesis naturally uses guideline(3), but this is obvious only tn the route to (11) and (12), the choice of conditiuus for e.g. tne synthesis of inter mediates (2) and (21), and in the reagents chosen for the synthesis of (3), (8), and (15) 98 Every synthesis starta from available materials Note particularly that alcohol (5) appears often in these pages, and that the simple ring compound, (10) [= (13)] appears twice, The details of the synthesis of (4) would be decided largely on this guideline. geanple ; The substituted 1,4-diol (4) must surely be made by a method which uses the symmetry of the structure. Disconnection (a) would require Grignard reagent (5) whose OH group Would have to be protected. Disconnection (b) gives hydroxy ester (6) and here no protection is necessary as the internal ester (lactone) (7) serves the purpos Analysis Ph yo,Pd Ph Ph 0, a, ~ oo > a ° Mype o a) (6) a «s) synchesiat?® Lag ,Etg\ Pabr —————> TH(4) 2) Peoblem + Suggest syntheses of Ts (0) and (9), using the symmetry to guide you “ OOO «yt t . Anever : Disconnection (a) on TM (8) gives symmetrical ketone (10). The alternative disconnection (b) would require bis-Grignard (11) ~ a doubtful species: = LO + BtMgpe Om@ ae > EO vw Brilg MgB ap Ketone (10) (fluorenone) is available and route (a) bas been used successfully Syntheote?®? Eeagbr (20) #5 ms) 73% ‘The same two disconnections on TM (2) Suggests (b) asthe better route. Grignard reagent (12) might be difficult to make, but ester (13) is a simple 1,2-dix problem. snatyote a Mbr = J + apt Puc he 12) a OP i bi , a cas) capren br Pom ogee Syntheate!™? ArveBe L.base _PhOH reo, FO 18) > 9 BOUL i e.g.65%, Ar=p-Tol Problem : Alcohol (14) was needed to make the corresponding Grignard reagent.'®* Suggest a synthesis guided by branch-point disconnect ions. tnoven + Dipoonnoetion back te the firet branchpoine (@ in 14a) requires formaldenyde and halide (15) which jon back to the cones from alcohol (16). Disconn second branchpoint (# in 16) gives two available starting materials. ‘Br 2 4 Ade . a6) The synthesis has been carried out by this route"? there is an alternative for the first step which gives 8 higher yiola.1#* Synthesis ’®®)1* tale Eo) 46% PBry Lt — ss tts mncray 61% 2,cig0 30% 1alte 2 1» > 18) 2MeCHO” ‘gre 102 ample : AS purt of @ programme!!? ta serean for new perfumery compounds, alcohol (17) was wanted, This has tuo branchpoints (@ in 17) and disconnection between them simplifies the problem a great deal. Ketone (18) js an available natural product (thujaketone) and nalide (19) can be made via a Grignard reaction. Anatyet 1,1 ¢-c on an Bn * “ as) 4 HO, HO Lice = (20) + n-Hexbr mthoasal)® Lg Br scwoxpe “3 ___ (20) 2 sy 2A-PRCHCHO 73% 60% 1M = mc17) 1 BB) ee The product (17) had only s mildly ploasant nell and was not adopted by the perfume industry. Eeample : By contrast, the synthesis of the potential anti-malarial (21) uses no disconnections at branchpoints wank (aay The amine could be put in by reductive amination on the secondary side or via an amide on the primary side (Chapter 8) but thie crowged compound can be made by simple alkylation of (22) with available iso- Propylamine. Steric hindrance prevents a second alkylation. Analyote 1 . waren Suk a CoN (ata) (22) The OF group in the required alcohol (23) is too far (cine atoms) from the branchpoint so the best we can do is use an epoxide to give half ether (24) as the starting material. Analyete ° (22) => 00 A = a (23) . we 0 se Yr (24y This looks at first sight 1ike a formidable problem of chemoselectivity, but changing the oxidation level to an aldehyde (25) gives a simple 1,3-diX disconnection to avaliable aldehyde (26). The ester vould nave done as well. Amaiysie 3 FoI HO 1 g-aix ie nn a ‘0 5) a (36) Syntheota’*® Meo” i, SOCIy (26) SS (23) > (21) ——> 1 Ye a ar % 908 A 92% soc, ‘Ny Late oe —> (22) ——> mar Ly 8% 908 88% Yroplem : Suggest a synthesis of acid (27) needed to confirm the structure of an unknown compound.??® Con «en ver: Tae Grignard method will be best for making (21) from secondary halide (28) as it avoids elimination reactions likely with cyanide ion. Disconnection (20a) now leads back to an alcohol (80) which can be made in a similar manner as (24). alysis . a (27a) (28) v owe = iw en oO woo, (29) cl FGI BN 0 — uw Hoo 0 Tetuction (30) V Meow + SHO Syntheste?!? - NaBH, Meo) Ko 4 Q0 > eo 80) S0CLy 2g M007 Ne 09) > 2.ELCHO 705 19% SOCly 1M > 28) —— > m7) PYF 1% 438 2.00, CHAPTER 12 Strategy V: Stereoselectivity A ptieatly Aetive compounds Sroblem : The sex-pheromone of the rove-beetle (1) can be made by the following sequence of reactions. Optically active (1) is required for biological activity. How would you obtain it? syntheste®*® on Le p-OetBe ——————> poet AN 2 0 trex Avon _ Teo [oe HO" /11,0 Veorene n-0et ’ a sever : A resolution is necessary and the alcohol (2) Ss the first compound which has the chiral centre (e in 2) so this was resolved by combining it (as a urethane) sith an available optically active compound, on "C01 (2) a-Oer 107 Feampte + The discarded (-) phenylethylamine (3) from the resolution on page T95 can be used to make other optically active compounds. HN “te My Ny (-)-(3) (4) ) ‘The strange amino-acid (4) is a ‘fat’ version of phenylalanine (5) having a side chain which is rigid and inert, but which is also space filling rather than flat.t?* optically active (4) was needed to study peptide conformation and the biological activity of drugs. A Strecker (p T 4g) synthesis would require aldehyde (6) which can be made from available alcohol (7). We shall now use R for tho adamanty1 group. Anat (6y=n ‘The one-step Strecker synthesis offers no opportunity for resolution and (6) is achiral. However by using available (-)-(3) in @ two-step Strecker synthesis, asymmetric induction can be used to make only the wanted enantiomer of (4). The PhCilfe group in (8) provides a chiral environment so that cyanide adés rreferentially to one side of the C=N bond. Crystall- isation of the major diastereoisomer of (9) (note = chiral contr (e in 9) und hydrolysis and hydrogenolysis give pure a). ensures the integrity of the new centr: The (-)-(3) is recovered und can be used again Pyntheate®® on uso » eo ——_ im = ‘ om CF,COgH (Readamanty) joc” a on al Hew x ~ Exo we py i cone @) 78% erystallises out st enemy pace aN, > mi 6sy-(4)-a) aa 2.cone. HCL Phi Feomple + Oprieally active aldehyde (10) wae needed for a synthesis of biotin.'!* The compound has a 1,1-dix disconnection (10a) clearly available and a C-N (amide) @isconnection leaving (11) which has the same skeleton as the amino acid cysteine (12), M27 co Phi ao) Anatyete Meo,e MeOg¢ cH0 cH is 1,1-dix lis ot => pheno + us roa) ‘ coe em i, yo i, yy ‘HS’ us a2 an It is better to add the benzaldehyde and the COjMe group first so that conversion of COjH to CHO does nat affect the reactive NH, or SH group. Note that diborane, Bglig, reduces COyH even in the presence of an ester. synthesta’** he CO,H PacHo Kafecost ay Ss ok s __eiconte Me Bolly neous dry THE base we0oF 0 cro, mJ —— > mao) pr Problem Suggest a synthesis of optically active S-(+)-suleztol (13), the aggregation pheromone of the wood-boring ambrosia beetle, from available ethyl (S)- (-)-lactate (14). ou (8)=(4)-13) Answer : The required disconnection is (13a) which cloarly needs optically active cpoxide (15). This muot be made from (14) without inverting the chiral centre so reduction of the CO,Et group and conversion to a leaving group are needed. Ana’ reduction as) The chomoselectivity of converting one OK in (16) into a leaving group will be easier if the other OK is protected, so it is best to introduce protection at the start. The easily removed THP group is ideal (Table 19.1). The required Grignard reagent is from available halide (17) (pT 1). Synti orke at dee LiAlky anys — noc — iP . wn _TSCL {eo ® on. wo Jet 0 ke no Ue" PvE Meo ow BES (s)-()-128) 45% from (14) 1g ——_— 68)-14)-118) el 2.(8)-(-)-(15) 76% Epoxide (15) has become an important intermediate in chiral syntheses. How would you make the other enantiomer (R)=(+)=(15) from (14)? Ansver ; An inversion ie now necessary, so we must make the tosylate of the secondary OH group before reducing the ester. Diborane will reduce the ester in this case. Syntheeto'3® ors Tse le! Bs base (14) —> rro,¢ ae pyr (-()-5) imple : There have becn warious attempts to generate optical activity without the investment of some optically active reagent. The most notorious is the alleged synthesis of optically active santonin (17) from in- active precursors.'?" This was quickly exposed as it transpired the reactions didn’t even give the right products, let alone optically active ones.!!* In more modern times the epoxidation of (18) in a ‘confined vortex’ (14,000 r.p.m. in a turbine) has been reported!?® to give a product with a tiny optical Yotation, but doubt has been cast on this observation too. #4 won e Hy We as) a7 So far, the methods described here and in the text are the only ones which work : optically active starting materials must be used, or asymmetric induction with an optically active eubetituent or reagent, or the product or an intermediate mst be resolved Steveoapectfic and Steneoselective Reactions Problem : In the synthesis of a chiral crown ether,’ ** compound (19) was needed. Suggest a synthesis for it APA AY anever : Two 1,3-d1X (C-0) disconuections (18a) reveal the ‘ran diol (20) as the vital intermediate. This can be made from cio epoxide (21) by stereospecific inversion sg PA ga (aga) 2 ool anaix "Opt ad oH + SAN yo wel cy Synthesis??? \S/ serps Hor a > (21) —> (20) ——> TmAI9) 1,0 Meo" MeOH loblem : Why dothe base catalysed reactions of (35) and (37) in the text (p T102) give such different products? Answer : This cia compound (35) cyclises because the anion (22) is held, by the almost rigid five membered ring, in the perfect conformation for stereospecific Sy? displacement ™ 07 ,Cily,OTs 1inear ‘The trans compound (36) fragments because cyclisation is impossible and because it, unlike (35) can adopt the perfect W conformation (all anti-peri- planar bonds) ideal for fragmentation (23) (23) Problem : Two of the following reactions are stereo- specific and one is stereoselective : explain which is which and account for the results. base (28) (en) Ameer + The first reaction is iodolactonisation and is a stereospecific trans addition to the double bond. The iodine adds to form intermediate (28) which opens by Sy’ inversion (arrows) to give (25) “0, ( A 1 (24) ——> Ye —> es) Ratico, The second reaction is a stereospecifically trane elimination (£2), The marked proton in (25) cannot be lost as it is cfs to the leaving group (I). Trane elimination (29) is possible only to give (26). aay, Th — (29) ‘The third reaction is the epoxidation of a double bond which ean take place from either side. The per- acid attacks from the less hindered side away from the lactone bridge (30). This ie the stereoselective reaction. Cn Lo > en (30) COO Epoxide (27) has been elaborated into useful synthetic intermediates.*?? Problem : Suggest a synthesis of (31), needed to make prostaglandins."?* (a1) Anower : This compound (31) is nearly a ring-opened version of (26) so a similar sequence will be satisfactory ° ° coat y j Ig base MeOH —>-_ %,, Natio, t TH(31) Conformational Anaiyete* Seample : Grignard reagents do not normally react cleanly with alkyl halides to give hydrocarbons (32) RllgX + 8'Br > RR! because of metal exchange, radical reactions and elimination, However, Grignard reagents add cleanly and in high yield to a-haloketones such as (33) to give the coupled product (34). * also involved in the Inst two probleme. Ph Pnitete (33) «aay This reaction is not what it scoms : work!?* at low temperatures reveals that an intermediate (35) 18 rorned by addition to the carbonyl group and that this re~ arranges to the product. Ph cl Phitgpr heat (33) ——> — ) 08 % (35) 89% Note that (35) mist be the stereoisomer with Ph and CL emane for the anti-peri-planar arrangement required for the stereospecific rearrangement, and so the Grignard addition must be highly stereoselective. This all makes sense by conformational analysis. Chloroketone (33) prefers to adopt the axial con- formation (38a) because of dipole repulsion between ¢-€1 and The less hindered side of the carbonyl group is therefore ¢razo to the chlorine giving (36) in 1a) - trang diaxial - for re- the right conformation arrangement. a cL PoigBr —. - —> 04) Ph (38a) (38a) Problem : Cyanide ion addition to (36) gives? a 2:1 mixture of (37) and (38). Explain. (The reagent and dipolar aprotic solvent simply ensure no confusing hydrogen-bonding. (36) MegN.cHO az) (38) 2 1 anaver : Conformational drawings of the products show that the cyanide is ariaz in both cases, Ler by cy (stay (38a) ‘The reaction must occur by axiai addition (39) to give (40) which may be protonated on either side of the double bond, but gives the all equatorial product (37) stereoselectively (39) (40) CHAPTER 13 One-Group C—C Disconnections I: Carbonyl Compounds (a) Carbonyl compounds by 1,1 C-C Disconnect ions Eeamplee : 1. On page 94 acid (1), needed for conversion to ketone (2) problem under investigation was steric hindranee in nucleophilic additions to ketones and a crowded ketone appeared a problem on the synthesis of The with a chiral centre was needed rr AW — a ~ The conversion (1) > (2) was carried out by the organo-cadmiun method (p T 106 ). 123 2. Among the many anti-malarial drugs tried!!! in the 1940s and 1950s was (3) (ef p 104). ‘The better C-N disconnection is towards the middle of the molecule, revealing Ketone (1) as @ useful intermediate. c-c disconnection is again preferred towards the middle of the molecule and leads back to available (5) as starting material Anatyeie Ux SY a 100 SH we > q (3) TT (coy eee co Bx eo Ye + Bee @ Bromide is displaced more easily from (5) than chloride. The second of our two methods, Grignard attack on a nitrile (pT 107), is used here. The final amination is carried out by catalytic hydrogenation of a mixture of amine and ketone. syntheste Moo” AN Lait (3)

neo’ cL MeOH 2.bteN @) ————> 1 ms) 54% Nyy 40% Probleme : 1, Suggest a synthesis of (6) found in lavender oil to which it contributes'?’ a ‘remarkable freshness’. «sy {neuer : Disconnection towards the middle of the nolecule provides 2 simple synthesis by either of our methods, Another alternative is preliminary FGI to alenhol (7) followed by a Grignard disconacction. inalyet OCL + CACC SH > 1 (Ga) cy + Csi Mabe Fet[fosierson a The synthesis has been carried out by the nitrile Synvheoset®® 1g aCe}, 8r ——> 6) ° 2.EtcN 61% 126 (b) Cambonys Compounds py ALkylatton of enots eeanptee ; Direct alkylation of nitriles is possible if a strong base is used (often NaNH,) : acid (8) can be made this way. Analyste FGI hydrolysis, pa~ CO,t pr cn (8) «a) L.NaNHl,NHy(1) kort Pax —<——>_ (9) ——> mH) a or goz 1,0 82% It is also possible to fill the last place in an otherwise blocked carbony] compound since further alkylation is impossible and the molecule is too crowded to condense with itself. Again strong bases are needed, such as PhC” used in the alkylation’?* of ester (10). 1.Phgoha on TB (a0) OBt 58 Problem ; Suggest a synthesis of ketone (11), used in the manufacture of phentermine, a drug used to control obesity. a7 aren aay tnover : This is a fully blocked ketone so we can remove one alkyl group, and we shall get the greatest simplification as well as the best reaction if we remove the benzyl group (11a) ° 9 FC Pat bh => Ph => Put + cr (ay Son ° ACly 1.NaNiy —> ttc) 2.PhCH,C1 76% Simple Probien 1 Suggest oyntheees of the perfumery ketones (12) (carnation) and (18) (gardenia). ° ° wy JOA, as) aa mo 4nover + These are ideal Ts for alkylation of ethyl acetoacetate (15) since 1,2 ¢ disconnection gives acctone enolate (14) for which (15) is the synthetic equivalent (pT 108) analysis ° PR™ 9 (osy Bu ay Aout 9 1,20-¢ (a) as) me cgay + RBr ‘he alkylation?#? and decarboxylation??? methods deseribed for the synthesis of (12) are typical Syntheste © Aromrzst 207 1.Na0b * ae —— m2) (5) CoEt 2ut’jneat °* 72% ample : Optically active acid (16) was needed (pT 107) for the synthesis of am ant alarm pheromone The branch point (@ in 16) is als the chiral centre so it is better to avoid disconnections there, The 1,2 CC disconnection (16a) is ideal as it gives synthon (17), for which we use a malonate ester, and halide (18), available from optically active alcohol (19), a major by-product from fermentation. Anatyete + CHC GH 2 004) a7) )-(8)-(19) ()-(8)-18) 60% Eto" 0,8 C4, (CO4Et) ——> A 2.8) ogbt 788 1 HO" go > 0-8)-18 2.8" heat 85% Suggest a synthesis of amine (20), needed to study??? whether cyclisation would occur during bromination of the double bond. Nit, vs (20) 130 Anauer + This hranched primary amine can he made from ketone (21) vin the oxime (pT 63), A.1,2 C-C dis- connection on (21) is good as it needs the symmetrical allylic halide (22) Analy Nou FGI cn oo) —=> —_ reduction ‘oxime ° a © 1,2 C-C = + CO,Et (a (22) A vroypr tase 1. KOH 2.(22) 2." heat ii,OHC1 Kow Liat, ay ——> mao DoF Eton . 51% Aa 85% ‘The cyclisation experiments were disappointing’** some unsaturated amines did cyclise but many, including (20), were simply brominated on the double bond. 2. A further study on eyeligation reactions? *? needed unsaturated diol (22). Can you suggest a synthesis? ru HO OH (23) Anewer ; ‘This is clearly a reduction product of a malonate ester (24) (ef p T110 ) which can be dis- connected in the usual way Analyste o ME PoE (22) (24) Tt will be better to add the more reactive allylic halide (22) in the second more difficult alkylation Syntheeto! 7 1.BLO™ | 9.810 LaAliy Clig(COsEt yy > ———> (24) ———> am23) 2.EWBE 4,(22) a8 ‘This time the cyclisation was more successful, the five membered cyclic ether (25) being formed in excellent yield in acia. asa “ (23) ——> i 8) (e) Carbonyl Compound Syntheote by Michael addition sample : One wdvanlage of this route is that it allows the addition of an ‘angular’ alkyl group in compounds like (26). Many natural products, such as steroids, contain an angular group and it can be difficult to set up this quaternary carbon atom. analysis 1,3 C6 > sore 0 0 R (26) @r Syntheote'** RugBr (27) ——> = m1028) cu(oae) ae Problem : Suggest a synthesis of ketone (25) (28) substituted side and (a), (b) and (e) are all possiti+ Disconnection (c) has the advantage that it is betwee two branchpoints and so gives two simple starting materiais jnatyote abe Ls cc a ayN SS rw" \ oO © ° (29) (28a,b,0) A Grignard reagent can be used for (29) without the usual copper catalyst as it happens. Syntheciet** i-Prgbr «SOT ° 80% CHAPTER 14 Strategy VI: Regioselectivity (a) Regioaelective Atkytation of Ketone mmpie : There are alternative strategies to the one outlined in Chapter T 14, Acid derivatives can enolise on one side only, the enols can be alkylated (p. 126) and conversion to the ketone achieved by one of the methods from pages T 106-8. For both steps nitriles are ideal functional groups. Hience ketone (1) might be made by regioselective alkylation of (2) but this is doubtful. A safer route is to disconnect the ethyl group to leave nitrile (3) which can certainly be made by alkylation of nitrile (4) as there is only one site for enolisation. YT py « as4 Pa => Px7 cw) @) cap) a (8) pen + PHCHET Pat Alkylation of nitrile (4) needs only moderately strong base (hydroxide will do!'*) as the benzene ring helps to stabilise the anion (5). nbhewtet "19? nN Kao prer Eolger ~w — > (4) ——> a ow @ | 50% t mur) Optically active ketone (6) was needed for a study of asymmetric induction.!*? It could be made from acid (7) by a Friedel Crafts route or from nitrile (8) by Grignard addition, but neither of these compounds could be made by alkylation as the branchpoint is on the 8 carbon (@ in each). The 1,3 C-C disconnection, e.g. (Gp) is not good as it destroys the chiral contre. Ki => Ie to * cog or Ph’ Ph Phi 2 Phi a “ © oy ° ‘The compounds could be made by alkylation if the cyanide (8) were derived from (9) and hence from cyanide (11), Analyete 2 wae Lon Sn toe =, Sa =, (8a) @ pwr => =>) Cok pH CK 2 Pat FGL —— reduetion ox Phe (a0) ay ‘The chiral centre first appears in cyanide (11) but the acid (10) is theideal compound for resolution as it can form a salt with a naturally-occurring optically active base. Suntheoso?*?/1*% NaNiiy HO" /H0 px ex) ——> a) —— Ber a7 L.resolve 2. Laan, LiAllly acy, R* ors, 3.TSC1, py? Pr’ puso RoC) Phuger Re(+)-(8) ——————>_ THE) RCH) Tosylate was preferred as a leaving group in making cyanide (8) and the Grignard method for the synthesis of (8) to avoid an extra step inpte Prob; en: Suggest a synthesis of ketone (12), analysing the possibility of using alkylation. Anower ; An alkylation disconnection (12a) is indeed possible but regioselective alkylation of (13) is not. We could make (14) by me hods to be discussed in Chapters T 19 and 20 or we could revert to mitrile (15), as in the two examples above. ery Anatyets 1 a Nw no good ° (as) (12a,0) cogEt ° (4) A, ae As (as) + aePratypr Easier solutions are to make (15) by cyanide dis- Placement or to use a third disconnection (12¢). Aen = 2. New o (120) ‘The synthesis has been achieved by this last method, using an organo-cadmium reagent. 1M Bt go a-Prcoc a-PrBr ————> i-Prca ————>_ TH(12) 2. 60% Provien 2; Cwmment on the feasibility of using die- connection (b) on TH (16) (p 192). b o as) Anower : We should need keto-ester (17) for this, and even then alkylation with secondary halide (18) is Likely to be poor. Alternatively we could use nitrile (19) but this requires the same alkyl halide (18). The Michael synthesis on page 133. is best Anatyato cOsBt ab ’ 2 + Br, > 0 (16a,b) az) cs) “y > ak => week + (8) giovelectivity Probion : Identity the factors responsible for regio~ selectivity in these reactions @ Me,Culi 708 ——> * (b) ow so CN pr cHo ZN nr, © Sh : : coyee Et Anover : (a) Steric hindrance equal at either site copper is the metal to ensure conjugate addition.'** (b) Kinetic attack by basic nucleophile on less crowded site.?** (e) No copper; Grignard attacks ester directly and displacement of EtO” is irreversible."*” Problem : Make the first disconnection to show how these molecules might be made by organo-copper addition to table carbonyl compounds. ~@ «) (20) to the carbonyl group. It is best to leave the ring intact and disconnect at a ring-chain junction (a-e) or at a branchpoint (4) Analyste Bh 0 u cos => + ene HO cigay RO cay ° © + Me 149 @) (20a) cay + ate" (25) Phy ° a8 hi - «ay AD > + Ph (22a) (26) Synthesis : ALL by RMgBr + Cu(1) + enone, Compound (22) was used to synthesise some central nervous system stimulants,'*! the others mostly in investigations of the stereochemistry of the reaction What generalisation can you make oa the st of the organo-copper additions? Anover ; ‘The reagent approaches axially in a tranc manner to the largest substituent ring B in (23), Et in (24), and t-Bu in (25). In (19), the last proton is added to enolate (27) to give the more stable crane cochemistry ring junction. bu tO PhllgBr (23) ————_> > as cut) H «2t) Example : Sometimes copper solves other regioselect~ ivity problems. Addition of aryl Grignard (28) to enone (29) gives the anomalous product (30) in which the electrophile (29) has been attacked at the right atom but the nucleophile (28, arrows) has attacked with the wrong atom, Mac Meo, Meo. (30) Addition of Cu(t) to the reaction mixture provente this unwanted reaction and the normal 1,4 addition product (SL) is formed.?*? Meo, cat) Further Ezample + Regicselsctivity in Epoxtde Re: We have already discussed the regioselectivity of the reactions of epoxide with nucleophiles and devised strategies (p 64-5) to achieve the synthesis of os wy A So A . (a2) Al Another way to make (32) is to carry out the reaction in acid solution when the regioselectivity is reversed AY“, as . Ms a a loose transition state so that a partial uponldy gives cation (99) which reacts » positive charge appears on the carbon atom under attack The more stable partial cation is therefore formed Strain in the three-membered ring plus the excellence of the leaving group make the C-O bond start to break before the C-Nu bond is fully established. Not all nucleophiles are compatible with acid conditions, and unfortunately most carbon nucleophiles, especially lMgbr and Ria, definitely cannot be used in crafts reaction, with an aromatic this way. The Friedel ring as the carbon nucleophile, is quite satisfactory. Os uw Pw CHAPTER 15 Alkene Synthesis Sy Elimination from Aloohole and Derivatives Simple Probleme : 1, Suggest a synthesis of alkene (1), needed for a morphine synthesis. *** a ‘One Amewer : Alcohol (2) will dehydrate to (1) and the Grignard route gives (2). Analyote ~ D ° Xs a o> => a one aenyan- ‘oe Bet C-€ Ot Me ° “ “Oo, 14s Halide (3) is available, but could be made by bromination of (4) or methylation of (5). In the synthesis, oxalic acid (6) was used as the dehydrating agent. This is a reasonably strong acid (pKa 1.23) and is conveniently solid. Syntheatat?® ao oe ——> ma) 78% CH 100% ° 0,8 6) Suggest a synthesis of alkene (7). ~ » ‘The OH group is best placed at the branchpoint to give alcohol (8) Anower : any arm of this symmetrical alcohol can now be disconnected. Anaty vot Sy ee ima VO Tree @) yntheotel®® Latig (6) Ewer ———> (8) ————>_ ™7) 2B.Et,co gay OATES se Feamptee : Unlncs the alaohal fe eymmatrical, of regioselectivity in elimination occur. ‘The general rule is that elimination in acid gives the more substituted double bond (e.g. 9) while elimination in base gives the less substituted (e.g. 10).!5* Ts0ll —= ~~ Y i) ‘Tech pyr t-Bu0- ~~ s YN cor? ors (19) 826 (418297) This may change with extra conjugation ; the benzylic protons (marked H) in (11) are more acidic than those on the methyl group so elimination in base!*” gives (12) not (13). KoBu-t uae NF 1-Buolt (12) as) ap 98-100% ‘The large base t-buO™ helps to give the less substituted alkene (10) and also helps elimination rather than substitution, as these contrasted results show'** for (14). Weo- Soe 1+98(18)1 Woon as) oe NAN (ay t-Bu ears Na [41282591 t-Buol (16) 858 probion : Predict the elimination products fron (17) and (48) under the given conditions HO Ph HCO gH —>> heat a7 pyr heat co cas) sewers Compound (27) seacts via carbonium toa (20> aud gives'®® the most substituted and conjugated double bond, i.e. (21). The product was used to make a bl- phenyl for studying cyclodchydrogenations and the starting material (17) is available from (19), made on p iis. e 1, Ph peMeBr Ph at “a — (as) (a7) 85% + pn Pa = (20) ey Compound (18) eliminates'®? in base with the loss of the more acidic benzylic proton (22) ef (11) = also giving the most conjugated product. Phe ? kes Pat (22) Beample + Special bases DBN (23) and DBU (24) are exceptionally reactive in elimination reactions under mild conditions. DBN allowed elimination of HBr from (25) even in the presence of the epoxide so that mono- ‘epoxy-naphthalene (26) could be made for the first A =“ OU 2 (7) iso-saffrole used in soap perfumes. Either or trans will do en auen + The Wittig disconnection suggests two sets of starting materials. wittig Ho => + Phe! c27ay (28) (29) *pphs cor = OO" (a1) (a0) Aldehyde (25) is piperonal (see pT 9) which we made from halide (31), the precursor of ylid (30), so either route is suitable. ‘The route via (28) has been used syncheote Btpr a - BuLi PhP > Psp Er Br > os (28) (23) ———>_ may 37% This route should give more fo (27), ac the ylid (29) is unstabilised, while the route vde stabilised (30) should give more trane compound. Prample : Ester (32) was needed as a model compound to study mechanisms of biological ester hydrolysis.*© It is clearly made from acid (33) which could be made from aldehyde (4) by a Wittig reaction. We dis synthesis of (35) on page 21. ed the Anatyete Meo 0, woe OF 2 = + Ko «ass enter 33) {prs ou "Ao ¢ Mi co clo (35) = + PagPL AH ether 34) ‘The methyl ether was introduced after the Wittig reaction which used a protected carboxylic acid (ethyl ester), p-Nitrophenol is available and is one of the products of the nitration of phenol. Syntheste?® E107 P+ BroH,co,ke ———> Ph,P*cH,C0,Bt — = HO CO,Bt 1.Me9S0, Nak 1.socl —> 033) >> 432) 5 of 2a ay Sor °, 2 ‘The double bond in (92) is crane since the Wittig used a stabilised ylid and an aldehyde. Examples : 1, Unstabilised ylids usually give more o¢e product with aldehydes. Muscalure (36) is a house-fly pheromone used to bait traps and the obvious Wittig synthesis gives 85% ¢ and 19% trans, Synthesis L.BuLi,DMSO_ MeCClly), (CH), Me ss + ae ‘aia! te —— MeCCH,);,CH,7 PPR, 2.Me( CH, ) CHO (36) 85:18 2:E 2. Diene alcohol (38) is the pheromone of the codling moth, the creature responsible for the grubs in apples, and of the various possible disconnections (a) is best as it gives most simplification and a stapilisea ylia (38) which will produce the required trans double bond Allylic bromide (40) and aldehyde-ester (41) are available, Analyete a wittig Sew 8) = (38) v fons AS, For co,F awa ia reduetion ay If we reduce the ester after completing the Wittig reaction, no protection of the aldehyde will be necessary. In practice, the hindered reducing agent (42) gives good results. Synthesie!®* PhgP 1.NaOMe, DMF (40) > DN rh, 2.(41) SRO” > 1028 . Bat waainyio oe}, Bf (42) Probleme : Pear ester (48) is an important industrial favouring compound with a pear-like taste and smell Consider all possible Wittig disconnections and choose a reaction which should give the right geometrical isomer C0 gMe — 2 (43) Anever : Each disconnection can be carried out in two directions : Analyste (44) a2 “0, Me => a ae (as) Phy (48) > See Eel? (43a,b) on cio, PosK ent a5 an “ “PPh, be (NS Goan => bo ‘The only reaction which should give the wrong isomer is (al) whore stabilisod ylid (44) should give an un- wanted yr» double bond. Both (b) disconnections should give the right isomer as stabilised ylids (48) and (49) should give a wanted trane double bond. How~ ever, the e¢s double bond in (47) and (49) is going to be difficult to make. Disconnection (a2) leads to the industrial synthesis as the half aldehyde, half ester (46) of fumaric acid (100% trang) is available and the Wittig reaction with unstabilised ylid (45) gives 85% cts geometry in the new double bond. syntheste?** . L.base Pad ty 3 2.(46) 85% 8,2 2, Bombykol (90), pheromone of the silk moth and the first whose structure was elucidated, can be made by two successive Wittig reactions, each giving the right geometry. Can you find this route? wf WPNen,) gctgon (50) Anewer : If we disconnect the eve double bond first (50a), we must use unstabilised ylid (51) and aldehyde (52) which can be made from stabilised ylid (53) and available half aldehyde, half ester (54) * *pen, PAN cig) 60! > yon mee wile qe (soa) oe os Sr arty geuig0H (52) yess cHo 1 ror ss (Ag) = S_ OWCL CH, CIty + OHC 0h, te ‘The synthesis has been carried out by this route and by many other routes, mostly involving Wittig reactions. Syntheotol®® (ite gSi ) N7Nal “s3) . (54) Dc HD 8D" 54% n-Pr bp hg baat nef NAN city) g00,80 ——-> (50 20% 66% Wenerae sample : Queen Substance (yd) Is a hormone produced by a queen bee to prevent the worker bees’ ovaries maturing and to ‘train! them to serve her ‘The obvious Wittig disconnection gives stabilised ylid (56) and keto-aldenyde (57). We have used many such long-chain dicarbonyl compounds in this Chapter and they are mostly produced from available alkenes by oxidative cleavage (e.g. ozonolysis). In this case cyelie alkene (58) is the right starting material, and this can be made from alcohol (59) by elimination Anatyete 9 Nitti BAW 0 SES (ss) > + (s7) 1a <= cc pawn + Mesigt (59) (58) ‘The Wittig reagent (56) is best protected as an ester and reacts chemo-selectively with the aldehyde rather than the less reactive ketone in (57). Mollet wH8O, 9 NajCO, PBPAAKR U0 ms) 54s CHAPTER 16 Strategy VII: Use of Acetylenes Example + The use of organo copper compounds in Michael additions (pT 118) has a disadvantage in that one of the two starting R groups in (1) is wasted. cut) — R RLi > acai ——> ry ea) a, retcans @ One way to avoid this is to have a mixed copper reagent (2) in which R is preferentially transferred to the enone, A good choice'*? for R’ is an acetylene and the preferred acetylene is I-pentyno (3) ae it is cheap, easy to make, and reasonably volatile. 160 Syntheate #7, 16# 1.NaNHly ,NHg(1) a nN = laa? (3) 90% R cu cut) 7 Li —s AA RLi Ze" «w ~ 5) Alkylation of acetylene to (3) goes in high yield, the cuprous derivative (4) is a stable solid, and the required reagent(s) cleanly transfers R to an enone, Problem : Suggest a synthesis for the sedative neparfynol (6). 6) Anowen + Amide disconnection gives ebloveformate (7), made from alcohol (8) with COC, (cf p 737). Alcohol (8) is a simple acetylene adduct. Anatyete Synthe, Probie : Explain the chenoselectiv: synthesis,27° of this no 1.Bu =/” ) 2.RBr Anover : Dianion (11) is formed from (9) with Bubi acetylene anion (pKa - 25) is very much more reactive than the carboxylate ion (pKa preferentially. the 5) so it reacts aay Problem : Suggest a synthesis for the starting material (9) and a synthesis for (10) based on a different strategy. Answer + (a) The simplest synthosis for (9) was alkylation of an enolate with available propargyl bromide (12) alternatives inelude Michael addition of acetylene to (43) or oxidation of alcohol (14). Analyste be « Sal. egos 8 et a2) smalonate et $ » FGI => — 8 Axo oxidation as) co,rt ook _2.KOH Ciig(COgL*)g ——> coger SK aco) 2.(12) 2.1" neat toa (b) The most obvious alternative strategies for (10) are the enolate alkylation (a) or the other acetylene disconnection (b). Analyete 1 b Sp eee + -exycoyt (5) snatonate ra + Aro8 PN A\0,8 The propargyl halide (15) can easily be made by acetylene chemistry. Analysis 2 on o> ee a= say 1.NaNHy .NHG(1) —_—~ Roe DBry Br Clg (COgEt)g —— as) toy OnEt — — > me) 2.8" heat Alkenee from sectylence Evanpie : Halide (15, RePr-n) was used by Butenandt!? in his original synthesis of bombykol (p 156) (16). rane double bond requires Wittig disconnection of the ete allylic ylid (17). Tt is easier to use acetylenic ylid (18) and half aldehyde (19) in this step because of availability. Bombykot ; Analysis Wittig SA] ENS as) 3 7 (Clg) gOH AD ett 4 ror ‘ ae one (a 90H oc ACB sO2Et 29) FGI “PP hs on Hey a Ly Br on y= = VEY Br => v= =—>wJ. (20) + cho Acetylene (20) was the one selected by Corey for use with organo cuprate (p 161).)+ NaOet , EtOH —_, Liala, 4 NAR Rly) EE — HCL) yplure (21) of the pink Froblen : The pheromone go: bollworm moth is a mixture of double bond isomers at bond (a). The eve double bond (b) can be made from an acetylene so disconnection (24c) is suggested, How would you make both efe and trane (23)? snatyats 2 (City ),0Ae FGI 2B > Bea (aly (CH) g0Ae = CH) gORC Anever : Acetylene (24) will give both by reduction under the right conditions (p 7127). Further dis~ connections give simple electrophilic starting materials n-Bubr + Reduetion of (24) by Lindlaar hydrogenation gives a o¢e double bond and sodium in liquid ammonia gives the trans double bond. L.NaNHy NH (1) 1.Bubt Sar aaa ean eet —— 2.0-BuBr 2 9 a 4 PngP-mr, a BL Pe gy ———> eis (23) Pec Wy Todi aar (24) PhgP.Bry \eey py ee rane pi oF (23) NHC) ‘The acetate group in (22) is inadequate protection for the terminal OH group and a THP group was used instead’?? (Table T 68), cuntheate 2 1 BuLa ==— cH) OTH 2.(23) mc21) row \ 1 tly. bindlaar \ Taeclace Y 2, heel, ACOH (Cll,) oTHP '¢ Advaneed Example : Lactone (25) was needed in the synthesis of pederamide, an inhibitor of protein synthesis found in a beetle, Disconnection of the lactone reveals two stereochemical problems : « e¢e double bond is required and two adjacent chiral centres (e in 26) must be set up correctly i (26) If the efe double bond were made from an acetylene, disconnection (27a) becomes possible, allowing stereospecific opening of epoxide (28). The trans epoxide is needed as the reaction goes with iaversivn. Anaiyeis 2 FGI (26) ——> reduction cs) “F ‘The published synthesis! uses (28) with acetylene itself, the acid group being added later. Note that the carboxylation is chemoselective, the dianion of (29) being the reagent (ef p T1290) Synthesis!” REO, wae 0) ase (29) 77% 1Buls 2.00, Hy 42> (27) ————— > 8) ——> TH) ant 562: Pad-C-BaSO, neat” g9@ quinoline Problem : Suggest a synthesis of diol (30). (30) Ho! ‘oH Anevor + The efe double bond comes from the acetylene (31) made trom two ketones. Analyste FOI a) ===> reduction oH ‘on — aye 2D In the published syatuesis, (he eyclopentanone is: added last, using the dianion of (31), No doubt the other order would be equally effective. ———_ = > o OF (31) —————> (50) 44% Syntheate Aetones from Acetytence : Alkylation of carbonyl compounds (pT 108) with propargyl halides gives y,6- acetylenic ketones. Hydration then gives a 1,4-diketone of a type we shall use in Chapter T 25 as in the following example.?7* co,nt co,ee CK ee SS ™. ae F0.Et MeOH BPs.Bty0 — C1gC.COgH 1. Compare these two routes to the simple ketone (33)!77 and the one uscd on page T 190, Which disconnection and which synthons are used in each approach? 1.Mg,Et,0 Betas (ret. 177) 8) Csi BY ——STyeco 3.cr0, 8 ° PN 1.to- sy 2.n-BuBr vy Shey Siwoniye ‘neat (Fels 182 and 133) c)acetylene route, see chapter T 16 Ngpr ey Syathe but with opposite polarity so that the acetylene anion es (a) and (c) use the same disconnection in (c) is a reagent for the synthon (34). 2. In the search for cortisone analogues, compound (35) on on (35) OH (36) ir was chosen for biological testing. Suggest a synthesis for (36) the precursor of (5). Anaver : Bromination of (37) should give (26) since only one side of the Ketone can enolise (p 753). ‘This Ketone (37) can be made from acetylene (38) and hence from cycloheptanone, ow on (oo) => => => Ss (38) ‘The hydration of (38) gave an excellent yield of (37) which was brominated in acid solution. = Br, bgt) 0 ——> (38) ——> (A) FT> C8) NaNily, WLHAO gs NH) Compound (35) was made by this route and did show some cortisone-like activity, but not enough to justify its development. CHAPTER 17 Two-Group Disconnections I: Diels-Alder Reactions Sinple Problem + Find Use Diels-Alder disconnections in the following molecules and draw the starting materials He Cope @ Se ° C058 BLO one Phy Ph @ o Ansver : Compounds (1) and (2) have only one double bond and one six-membered ring, so only one disconnection is possible. 178 + ‘0 0 co,Me ote D-a, = + Cope COgMe (2a) Anhydride (1), used to make polycyclic ketones??? and diester (2), used in a study of oxidative decarboxy- Intion,?*? have each been made in one step by Diels~ Alder reactions. ‘Two discomnections are possible with (3), but we prefer (a) which puts the electron-withdrawing COsEt groups on the dienophile and achieves some simplifi- cation. This compound was used by Woodward'"? in his synthesis ofmarasmic acid coset Bt Br0,0 oe” IX UX oc ore (ay Cone CogBt we (3b) il Both disconnections on symmetrical (4) should be carried out as very simple starting materials result.!** 6 ° Ph Ph Ph Ph (4a) nple Example : Lewis acids, such as AlCl, catalyse the D1eis-Alder reaction. orkers!*? used a three-fold excess of butadiene to react with the AlCl, complex of 5,6 and 7-membered cyclic enones, e.g. (5), giving excellent yields of ets fused bicyclic ketones (6). Me and H must be eie in (6) as they were ove in (5) oe 2 : (8) 6) Stereoapecificity. Problem : What stereochemistry is required in the starting materials for these Diels-Alder reactions:~ cho 9 a @® m Anever ; Diels-Alder disconnection (7a) reveals a diene (9), with no stereochemistry, and a dienophile (20) waich must be Crane to give trance groups in (7). ‘The one-step synthesis is successful,’** cio Ho pea, ' => (10) (7a) «9) Diels-Alder digconnection of (8) reveals a cyclic dienophile (11) in which Me and B must be cde since they are also cfs in (8), and the one step synthesis! *® duly gives of product. ; ~ - (as) dehydration i HOC. Dea 2 => + HO cog Ht as) The synthesis turns out to be remarkably easy. The crag adduct cyclises to ule strained anlydride (13) on heating with acetyl chloride. ar cH,coct (8) ——> 08) > a3) 150°C % ea 74% sealed tube 56s Syntheste eivity ‘The intra-molecular Diels- Alder reaction was used to make (19) ceded for a synthesis of the natural product torreyol.)"* The inter- mediate (18) could not be isolated as oxidation of alcohol (17) gave (19) directly. The intramolecular Diels-Alder must be very efficient, 4 H «sy as) (ay AN ‘The formation of (19) was very stereoselective, (19) being formed in a 9:1 mixture with (20). Both (18) and (20) are endo adducts, as diagram (21) should make clear: both marked H's have ta be cin if the CO is to be close to the diene in the transition state 8 6 i IE i » > > 2 a BY ¢ 5 cas) @ (22) ‘The i-Pr group can be in position a or > : it will obviously prefer to be away from the rest of the molecule at b and when (21) is unrolled as (22) b is indeed the preferred up position. oblem : The synthesis of bicyclic lactone (25) by the acid-catalysed cyclisation of (24) could be tried if (23) can be made by via Diels-Alder reaction. Cogit (23) (ay (25) How can (23) be made? Include a synthesis of the diene. Will the Diels-Alder reaction give the correct stereochemistry? Anower + Diels-Alder disconnection (23a) reveals maleic anbydride and diene (26) available by a Wittig reaction from (27) or more conveniently from alcohol (28). witig = 0+ PhgP=Ciy camp“ (26) = Fog 30> has been used for Analyote (28a) Route (b), using acetylide ion, the synthesis of (26) which did indeed give the right diastereoisoner of (23) with maleic anhydride.?** Syntheete ~ BZ Ho. = Hy => — NaN, Pa (28) 94% Nig(2) ——> 028) + — 3) Kuso, ~ 4gy 7 0 Phe usual diagram (29) shows that (28) is indeed ¢ rdo adduct : the two marked H's are ele in (29) and must be ofe In the produet (23), a ° (29) wh 2 A slightly petter way (than the wethod on p T137) to work out the regio-chemistry of many Diels-Alder reactions is to identify the most nucleophilic atom on the diene ang the most electro- philic atom on the cienophile and join them, Thus the Me,N group in (30) makes atom a (but not ») more electrophilic. correct ‘ortho’ Combining these atoms gives the Adduet (32) used on page T137 for the tilidine synthesis 0. Mei Pho OH (30) G1) (32) Advanced Ssample : In research on violet perfumes, aldehyde (33) became a key intermediate for a new series of compounds. Diels-Alder disconnection gives acrolein and diene (34) which can be made by the Grignard method from available enone (36) asa anal ade (34) Fo1||denyaration KH (38) Note that the Grignard reagent adds regioselectively to the CO group of (36) while the debydration of (35) is unambiguous: whichever Me group loses the proton the same product is formed Synthesta®®® uebigt at Frew 38) + [(s5]] —> ce ——> no) 38% 85% Problem : Comment on the high regioselectivity of this Diels-Alder reaction. Anover : All three Me groups in diene (34) direct the same way. The product is ‘ortho’ with respect to CMe, and CHO and ‘para’ with respect to the other be group. Alternatively we can argue that all three Ne groups in- crease electron density at the camo ond of tho dicne (37) and (38), (37) (a8) FGI on Dielendlder Adducta ample : ® family of biologically active compounds isolated from African trees include polygodial (39), an antifeedant which protects the tree against plagues of caterpiilars.!" Ho no cHo cuo) (39) (40) Direct Diels-Alder disconnection of (39) requires the absurd starting materials (40) and has the activating groups on the diene instead of the dicnophile The two aldehyde groups are in any case unstable and the Giester (41) would be a more amenable target. Tf the Soubie Pond were between the ester groups as in (42), a Diels-Alder disconnection begins to look good, and the addition of an extra double bond as in (43) provides a very simple Diels-Alder disconnection. 186 anatyete ror Co,Me oo) ——> OMe reduction => ii ay cogie Gopbe conte JcO,se => i 43) 4 (42) pr e056 co + Ss. co,tte (4a) (4s) acetylenic diester (45) is available, the synthesis of (44) is known, and the Diels-Alder reaction goes in excellent yield 110% “a+ us) —> as) 16 hours Tt now remains to reduce out one of the double bonds. This is straightforward as it is the uncon- Jugated double bond we want to lose and hydrogenation operates on thermodynamic principles. The catalyst approaches (43) from the opposite side to the Me group and Uherefore adds hydrogen underneath. ynthovte 2°92 G0,Me u COM (a3) > fi (40) ‘The problems of moving the double bond and getting the right stereochemistry at the third chiral centre in (41) turned out to be trivial ; isomerisation with HCI achieved both#. The final reduction was done in two stages with special reagents ner Lian, (46) ——> (41) ——> 808 (Cocl) = 9) DSO = * cwane decalins prefer a double bond in the position (41) as the otherring hus a better chair conformation The COpMe group 18 equatorial in (41) CHAPTER 18 Revision Examples and Problems There is no workbook material corresponding to Chapter T 18 so the opportunity is here to revise material presented so far Simple Revision Problems : 1 _ Suggest a synthesis for the perfumery compound (1). ( \- + er: Disconnecting the ester first of all gives an acid (2), made by straightforward acetylene chemistry (Chapter 16) and an alcohol (3) which is available, but which could be made by simple Grignard chemistry co gO o=> E ester, ANN a + C0, mcs? SS A AF 188 + HO’ m 'y br a+ ‘The synthesis of the acid'®? and the alcohol’**, 4nd the esterification are straightforward, Commercially amyl alcohol', a mixture of (3) and (4), is used. br La AL Vso on AA & 2.CHg0 i ill L.base O ia NN 2.CgH Br 1Na, @) —_— 8) 00, ie 2 2. (a) Suggest a synthesis of the amine (9). AWARD o nover + We wish to disconnect the ring-chain C-N bond but we must first go back to amide (6) to make the synthesis reliable (Chapter 8). tnalysie FGI (9) equction AA => Roca + oO w ° The reduction may be carried out with Liat ty uh ©) aD n-CgHly gCOC? chromite 928 (&) But how ean you make amine (7), when this approne! is impossible? pox) m Anover : The solution must be to disconneet the other two C-N bonds. This can be done directly or after FGI to double amide (8), when the anbydride (9) is a suitable electrophilic starting material. Br PaNiy (9) Tue Sveond UF Liese routes provides 4 reanuaubl) synthesis. «) LAATHy Phkil, ————> (8) ————> 7) 52K 1, As part of a programme of synthesising derivatives of branched acids, keto ester (10) was required. cosste (0) connection (a) is the best as it splits the molecule into two largish pieces and it allows us to use (33) tor the nearly symmetrical synthon (12) (see p 7 39), shatuete ? Opie van a2 a3) (10a) An organocadmium reagent (14) will react with an acid chloride but not with an ester (p T 199) and so oan play the role of epnthan (11) Meaknt (2) te Aten or Kye hy ek ‘ a3) — 1) br 2.CAC1y 13) > mio) 74% 2. Faced with the problem of making simple vinyl ketones (15) and (16) as intermediates in a longer synthesis, chomists'?? found they needed both strategies outlined here os woe + Ke on aa ° BS uke pve rT (15,n68t) b . (16, R-a-Pr) => mo Mee Bthyl Grignard added chemoselectively to acrolein (17) in good yield, but iso-propyl Grignard action, The alternative strat failed to give the r (6) was successful in Syntheste near Sy Be “+ Uy 2.17) cas mcs) 1, Frontalin (18), the pheromone of the western pine beetle, is an acetal (atom @ has two single bonds to oxygen). Disconnection reveals diol ketone (19). Analyste t 1 40 = oe 1 1-aix as) as) The aio part of (10) could come from hydroxylation of the double bond in (20). This looks 1ike an alkylation product of acetone, but the required halide (21) is not easy to make. An alternative is to make (20) from Grignard attack on nitrile (22) rol ae > hydroxyl- selon deMgBr + NC (22) Cyanide displacement on (23) gives (22) and we can make (24) easily by halide (23) ylation with symmetrical allylic = a (23) a This is a long synthesis for such a simple compound but every step is ensy to carry out. lydroxylation was replaced by epoxidation in the published synthesis. - P% yy LELO™ Fake 1 Kou o> so? 2.(25) oo,nt 2+ AcOH jaan, TsC1, pyr > on SS COgk cA, (C0, 196 2, In the search for analogues of tetracyclines, com pounds with sulphur in the second ring were investigated and (26) was needed for this work.!?? wo 9 (28) The starting material was thiol (27) whose synthesis appears on page 25. Michael addition to available (28) and hydrolysis gives diacid (29) which was cyclised in acid (iF) to (30). Removal of the chlorine atom and the methyl group guve (26). Synthoate)™* ne0,0Svo,Me sass 8) COgHt 2.HCL,H, Taco Opt one owe (@7) (29) 8am ch HBr OAc wed HOO (30) 96% 98% ontaiming antibiotic proved ‘The final sulphur. be a highly active compound, ‘superior in its anti- bacterial spectrum to all known tetracyclines’. Hore Advanced Problems : 1. In the synthesis of (26) ahove, why does the cyclisation of (29) to (30) give such a high yield? What is the purpose of the chlorine atom in (27) since it is not present in (26)? 4never : (a) Only one site is available for cyclisation and the two carboxylic acid side chains are identical, The only possible cyclisation product is (30). Reaction at OB is blocked by the MeO group () The chlorine atom blocks the alternative par: cyclisation which is favoured when (31) is treated under the same conditions.*?? CO,H uF S ed “COs! (31) owe Meo 2, G-Halo ketone (32) was required for a study of the photochemistry of this class of compounds. synthesis offering full stereo and regio-control.

You might also like